SAEM Tox, Infxn, Optho, Foreign Bodies, SAEM AMS, 2017 CV, 2017 trauma, SAEM MISC, SAEM - Shock and Sepsis, Environment and Endocrine, Pulm Emergencies

¡Supera tus tareas y exámenes ahora con Quizwiz!

Choose the INCORRECT statement regarding thoracentesis from the anterior approach (needle decompression): Answers: 1.The recommended insertion site is the second intercostal space, midaxillary line. 2.A 14- to 20-gauge needle is inserted perpendicularly over the superior edge of the rib. 3.After the needle is inserted into the pleural space, a rush of air confirms the presence of a tension pneumothorax. 4.If a tension pneumothorax is confirmed via needle decompression, then a thoracostomy tube should be placed as soon as possible. 5.An upright chest X-ray should always be performed following a thoracentesis to confirm the successful relief of a tension pneumothorax and the absence of hemothorax or other complications.

1

Following a motor vehicle collision, in which of the following patients is an emergency department Caesarian section most likely indicated, assuming a fetus at 29-weeks gestation? Answers: 1.Mother pulseless and apneic for 2 minutes' duration and still in arrest; fetal heart tones of 100 beats per minute 2.Mother with abdominal pain, blood pressure of 80/40; fetal heart tones 100 beats per minute 3.Mother with vaginal bleeding, blood pressure 118/78; fetal heart tones 80 beats per minute 4.Mother with severe head trauma, blood pressure 170/90; fetal heart tones 120 beats per minute 5.Mother with gunshot wound to abdomen, blood pressure 96/42; fetal heart tones undetectable

1

The blood pressure at which malignant hypertension is defined as present is: Answers: 1.an elevated arterial pressure associated with end organ damage 2.an elevated arterial pressure that exceeds the patient's baseline by 33% 3.diastolic blood pressure of 110 or greater 4.systolic blood pressure of 170 or greater 5.systolic blood pressure of 180 or greater

1

The chest X-ray in the Figure was taken in an intoxicated patient who is conversant, but an unreliable historian. The X-ray findings are best described as indicating: Answers: 1.Esophageal foreign body 2.Intratracheal foreign body 3.mediastinitis from esophageal perforation 4.normal chest

1

The patient depicted in the figure presents to the ED just after sustaining a pellet-gun wound to the right eye. What do the arrows most likely indicate? Answers: 1.hyphema 2.hypopion 3.iritis 4.keratitis 5.pterygium

1

What type of rhythm disturbance is seen in the EKG below? Answers: 1.first degree atrioventricular (AV) block 2.second degree AV block type I 3.second degree AV block type II 4.third degree heart block

1

Where are the normally dominant pacemaker cells of the heart found? Answers: 1.sinoatrial node 2.atrioventricular node 3.bundle of His 4.accessory pathway of Kent

1

Which of the ocular findings below is associated with hypertension? Answers: 1.arterio-venous nicking 2.cherry red spot 3.increased cup-to-disk ratio 4.retinal nevus 5.Roth spots

1

With regard to specific causes of hypertension, which of the following is true? Answers: 1.Hypertensive encephalopathy is more likely than hypertensive stroke in patients whose mental status changes are reversible 2.Hypertensive encephalopathy causes adverse outcomes over days or weeks, rather than hours 3.Patients with stroke syndromes must have blood pressure normalized as quickly as possible to reduce the risks of worsening neurological deficit 4.Laboratory analysis is rarely useful in cases of confirmed pediatric hypertension 5.Laboratory analysis is rarely useful in cases of confirmed hypertension in pregnant patients

1

A 26 year-old presents with agitation, chest pain and a heart rate of 142 bpm after intranasal cocaine use. The EKG is normal except for sinus tachycardia. What is the best medication to use in this situation? 1.Lorazepam 2.Diphenhydramine 3.Esmolol 4.Haloperidol

1. Lorazepam; Benzodiazepines are the treatment mainstay for cocaine toxicity. Lorazepam and diazepam can be titrated to treat the symptoms of agitation and increased adrenergic tone common to patients with cocaine toxicity. Beta blockers should not be administered due to a potential for unopposed alpha-adrenergic stimulation and resultant hypertension. Haloperidol and diphenhydramine can contribute to the hyperthermia common to patients with cocaine toxicity.

What is the normal range for intraocular pressure in humans (in mmHg)? Answers: 1.0-10 2.10-20 3.20-30 4.30-40 5.40-50

10-20

A 32 year old female is shot with a 38-caliber pistol at close range in the right anterior chest. She presents to the emergency department intoxicated and yelling. Her vitals include a pulse of 92, blood pressure of 134/84, and oxygen saturation of 97%. She has clear breath sounds bilaterally. The entrance wound is just above the right breast and an exit wound is noted in the right axilla. What is the most appropriate management of this patient? Answers: 1.IV access, portable chest X-ray, tube thoracostomy, and exploratory thoracotomy in the OR to search for cardiac or pulmonary vascular injury 2.IV access, endotracheal intubation and simultaneous placement of a right chest tube, bedside ultrasound, portable chest X-ray, and admission to the ICU if stable 3.IV access, portable chest X-ray, right chest tube placement if X-ray shows a pneumo- or hemothorax, admission to the ICU for observation 4.IV access, endotracheal intubation, CT scan of chest to look for pneumo- or hemothorax, or injuries to the heart or great vessels 5.IV access, endotracheal intubation, emergency department thoracotomy to search for cardiac or pulmonary vascular injury

2

In which of these patients is emergency department thoracotomy indicated? Answers: 1.Unbelted driver in a high-speed motor vehicle crash who loses his pulse while being extricated, and arrives at the E.D. after a 45-minute transport 2.Patient with a gunshot wound to the chest who upon arrival is unconscious and pulseless, with a systolic blood pressure of 60 3.Pedestrian struck with massive pelvic fractures who loses pulses and blood pressure at the scene 4.Patient with stab wound to the anterior chest who is dyspneic with an oxygen saturation of 80% and a blood pressure of 168/102 5.All of the above should undergo emergency department thoracotomy.

2

Of the following choices, which is the most likely diagnosis based on the EKG in the Figure? Answers: 1.left bundle-branch block 2.right bundle-branch block 3.anteroseptal myocardial infarction 4.Wolff-Parkinson-White syndrome

2

Of the options below, the therapy best for symptomatic 3rd degree heart block is: Answers: 1.lidocaine 2.transcutaneous pacer 3.atropine 4.oxygen 5.cardioversion

2

Using the rule of 9s, what is the approximate burn surface area of a victim who has sustained second-degree burns to the anterior chest and anterior area of both arms? Answers: 1.0.2 2.0.25 3.0.3 4.0.35

2

Which diagnosis is suggested by the EKG shown in the Figure? Answers: 1.digoxin overdose 2.right-ventricular ischemia 3.pericarditis 4.dextrocardia

2

Which is the most common associated neurological finding with a distal radius fracture? Answers: 1.Wrist drop 2.Decreased sensation over the thenar eminance 3.Weakness of finger adduction 4.Decreased sensation over the hypothenar eminance 5.Weakness with flexion at the finger MCP joints

2

Which of the following is an accurate statement? Answers: 1.Diagnostic peritoneal lavage usually cannot identify the presence of hemoperitoneum. 2.Diagnostic peritoneal lavage cannot determine the etiology of hemoperitoneum. 3.Bedside ultrasound can reliably determine the etiology of hemoperitoneum. 4.Bedside ultrasound can image the retroperitoneum. 5.Bedside ultrasound is the test of choice for diagnosing solid organ injury.

2

Which of the following statements regarding psychotic behavior is true? Answers: 1.Brief psychotic episodes, often precipitated by events such as death of a loved one, can be characterized by extremely bizarre behavior and speech 2.Delusions are defined as false beliefs that are not amenable to arguments or facts to the contrary 3.Delusional disorder usually results in impairment in daily functioning 4.Schizophreniform disorder is present when a patient meets the diagnostic criteria for schizophrenia but the process has been present for less than one year

2

You are practicing in a trauma center a receive a call from an outlying facility that they would like to transfer a male patient to you with a spinal cord injury after significant flexion and compression of the vertebral body. What does this injury pattern tell you about the patient's symptoms? Answers: 1.The patient likely disproportionately greater weakness in the lower extremities (as compared to the upper extremities) 2.The patient likely has paralysis and loss of sensation to pain and temperature bilaterally below the lesion 3.The patient likely has symptoms on only one side of the his body 4.Patients with anterior cord syndromes have only sensory symptoms

2

You need to treat an adult with no past medical history, who presents with a hypertensive emergency. You have access to all of the following agents. Which of the following is the preferred agent and initial dose? Answers: 1.clonidine PO 0.1 mg 2.esmolol IV 100-500 mic/kg load 3.hydralazine IM 0.1-0.2 mg/kg 4.labetolol IV 0.2-1.0 mg/kg bolus 5.metoprolol PO 10 mg

2

A 27 year old woman is brought into the emergency department by her roommate 30 minutes after ingesting a bottle of aspirin in a suicide attempt. Which of the following acid-base disorders is most likely to be present in this patient? 1.Respiratory acidosis due to somnolence causing decreased respiratory drive 2.Respiratory alkalosis due to stimulation of the respiratory center and increased CO2 production 3.Primary respiratory acidosis with compensatory metabolic alkalosis 4.Primary metabolic acidosis with compensatory respiratory alkalosis

2. Respiratory alkalosis due to stimulation of the respiratory center and increased CO2 production; key is 30 mins! few hours for metabolic acidosis

A 65 year old male is brought to the emergency department after he was found wandering on the street. He is unkempt and confused. A diagnosis of delirium, rather than dementia, is more likely if which of the following is true? Answers: 1.there has been a slow progressive loss of memory 2.the sleep-wake cycle is unaffected 3.there is a change in the level of consciousness 4.the confusion is worse during the day

3

A 65 year old male presents to the emergency department with chest pain. Cardiac monitoring shows a wide complex tachycardia. Past medical history is significant only for hypertension. His BP is 100/66, HR 144, RR 24, and T. 37.5. In addition to ongoing chest pain, he reports dyspnea. His level of consciousness is mildly decreased. Management should proceed on the assumption that he has what abnormal rhythm? Answers: 1.Sinus tachycardia with LVH 2.Supraventricular tachycardia with aberrancy 3.Ventricular tachycardia 4.Wolff-Parkinson-White syndrome with retrograde conduction

3

A patient presents to the ED after a fall with chest pain. A chest xray shows a rib fracture but no pneumothorax, and a chest CT is ordered. What is the most appropriate treatment for a small pneumothorax, detected only on chest CT, in a hemodynamically stable trauma patient? Answers: 1.Immediate needle decompression 2.Chest tube placement 3.100% oxygen 4.Heliox by face mask

3

An 80 year old nursing home patient is brought to the emergency department with an acute onset of confusion. Which of the following metabolic abnormalities is the most likely explanation? Answers: 1.hypocalcemia 2.hypokalemia 3.hypernatremia 4.hyperphosphatemia

3

Following a brawl at a local bar, a gentleman presents with an impressive right-sided periorbital ecchymosis. All of the following physical examination findings would suggest an orbital blowout fracture EXCEPT: Answers: 1.anesthesia of the right infraorbital region 2.diplopia with upward gaze 3.proptosis 4.right-sided epistaxis 5.right-sided infraorbital subcutaneous emphysema

3

Labetalol differs from propanolol in that labetalol is: Answers: 1.a mixed alpha1-agonist and beta-antagonist 2.a mixed alpha-antagonist and beta-agonist 3.an alpha-and beta antagonist 4.characterized by an elimination half-life of minutes rather than hours 5.selective for the alpha2-adrenergic receptor

3

The major abnormality on the image below is at which level? Answers: 1.C3/C4 2.C4/C5 3.C5/C6 4.C6/C7

3

The patient in the figure sustained minor blunt trauma to the eye, and has a normal head/orbital computed tomography (CT) scan. Ophthalmological examination is normal, other than the blood as shown in the figure (the blood does not cross the limbus). Of the choices below, which diagnosis is the most likely based upon the figure? Answers: 1.ruptured anterior chamber 2.hyphema 3.subconjunctival hemorrhage 4.globe rupture 5.foreign body

3

What does it mean to have 20/200 vision OD? Answers: 1.The patient's right eye sees at 200 feet what a normal eye sees at 20 feet. 2.The patient's left eye sees at 200 feet what a normal eye sees at 20 feet. 3.The patient's right eye sees at 20 feet what a normal eye sees at 200 feet. 4.The patient's left eye sees at 20 feet what a normal eye sees at 200 feet. 5.The intraocular pressure in the right eye is 20 and the intraocular pressure of the left eye 200.

3

What is the most common cause of death in Americans aged 20 to 40 years? Answers: 1.AIDS-related illness 2.Cancer 3.Trauma 4.Drug overdose 5.Pneumonia

3

Which factor is least reliable in differentiating between organic and inorganic causes of confusion? Answers: 1.acute versus chronic onset 2.Vital sign abnormalities 3.Presence of attention deficit 4.Signs of trauma

3

Which of the following cervical spine fractures is considered stable? Answers: 1.Flexion teardrop fracture 2.Bilateral facet dislocation 3.Transverse process fracture 4.Jefferson fracture of C1 5.Hangman's fracture of C2

3

Which of the following trauma patients can be managed conservatively without immediate laparotomy in the OR? Answers: 1.27 year old man with hemoperitoneum by bedside ultrasound; hypotensive 2.19 year old man with splenic laceration; peritoneal signs on exam 3.24 year old man with liver laceration; hemodynamically stable 4.30 year old man with a gunshot wound to the epigastrium 5.All of the above should go to the OR for exploratory laparotomy.

3

A 45 year-old is brought in 8 hours after a large overdose of his lithium. What is the best treatment method for this overdose? Answers: 1.Activated charcoal 2.Gastric lavage 3.Hemodialysis 4.Whole bowel irrigation

3. Hemodialysis; Dehydration, over-diuresis, and drug-drug interaction (particularly NSAIDs) are common precipitants of lithium toxicity in the patient chronically taking lithium

Severe lead toxicity can commonly result in which of the following clinical symptoms: 1.Constipation 2.Dermatitis 3.Memory loss 4.Stocking glove peripheral neuropathy

3. Memory loss; Lead toxicity affects a variety of systems. The central nervous system effects are many and range from encephalopathy and seizure to sleep disturbance and memory deficits. The peripheral nervous system can also be involved, with paresthesias and wrist drop being common

A young woman presents with an amitriptyline overdose. She is agitated and confused. In overdoses of this class of medications, an indicator of severe toxicity would include: Answers: 1. Elevated osmolar gap > 20 2. Metabolic acidosis with a pH < 7.25 3. Prolonged QRS interval 4. Serum amitriptyline level > 200 mcg/dl

3. Prolonged QRS interval

During opiate withdrawal which of the following symptoms would you expect to find? 1.Constipation 2.Pruritis 3.Tachypnea 4.Urinary retention

3. Tachypnea

A 65 year old male presents to the emergency department with palpitations. His heart rate is 250, blood pressure is 140/88, respiratory rate is 24 and oxygen saturation is 95%. The EKG shown in the Figure demonstrates: Answers: 1.ventricular tachycardia 2.a rhythm requiring verapamil as first line therapy 3.a rhythm that is difficult to identify with certainty 4.a rhythm which requires immediate defibrillation

4

A 68 year old diabetic male, previously living independently, is brought in by his family. He has been acting abnormally for two days. The family reports he is awake all night and sleepy during the day. He is confused about where he is and the time of day, and sometimes doesn't recognize his daughter and son-in-law. At other times he appears and acts almost normally. Answers: 1.Infection is an unlikely cause of his condition unless his temperature is > 102° F. 2.Dementia is the most likely cause of his condition and the family must be counseled about the future course of the disease. 3.Medications are an unlikely cause of this condition in the elderly. 4.Patients can be agitated and combative, or calm and quiet in this condition. 5.Treatment includes maximizing sensory input.

4

All of the following are true regarding adult epiglottitis EXCEPT: Answers: 1.Type b H. influenza is the most common bacterial pathogen to cause acute epiglottitis. 2.Adult epiglottitis is a cellulitis of the supraglottic structures of the airway including the epiglottis, vallecula, and the base of the tongue. 3.The incidence of adult epiglottitis has increased in recent years, whereas the incidence of pediatric epiglottitis has decreased. 4.Smokers have a higher incidence of adult epiglottitis. 5.The rapidity of symptoms' onset does not correlate with the likelihood of need for airway intervention.

5

The most sensitive bedside test for nerve injury in a finger after trauma is: Answers: 1.light touch 2.O'Riain wrinkle test 3.pain 4.temperature sensation 5.two-point discrimination

5

Which is not part of the Ottawa ankle rules? Answers: 1.inability to walk 4 steps at the time of the injury 2.inability to walk 4 steps in the emergency department 3.tenderness over the lateral malleolus 4.tenderness over the medial malleolus 5.tenderness over the talus

5

Which of the following is not a feature of febrile seizures? Answers: 1.generalized tonic-clonic seizure 2.duration less than 15 minutes 3.associated with a rapid rise in body temperature 4.occurs in children ages 3 months to 5 years 5.associated with postictal state of 30 minutes

5

Which of the following patients should undergo abdominal trauma evaluation? Answers: 1.22 year old with stab wound to fourth intercostal space on right 2.30 year old with pelvic pain and tenderness after fall 3.25 year old restrained passenger in high-speed MVA; no abdominal complaints 4.None of the above 5.All of the above

5

With regard to targets for therapy of elevated blood pressure identified during an emergency department visit, which of the following is generally true? Answers: 1.Patients with hypertensive emergencies should have blood pressure normalized (for age) within an hour or less 2.Patients with hypertensive emergencies should have mean arterial blood pressure lowered by 50% within 50 minutes 3.Patients with hypertensive urgencies are preferably treated with sublingual nifedipine, as compared with intravenous agents 4.Patients with hypertensive urgencies should have blood pressure normalized (for age) within an hour 5.The target systolic pressure for patients with acute aortic dissection is an absolute number rather than a percent pressure reduction

5

A 3-week old presents to the emergency department with a fever of 100.8. The child is otherwise well appearing. After blood culture, the best management of the options below is: A. UA plus urine culture, LP, antibiotics, and admit B. Check WBC before further management. C. Admit for observation. D. Discharge home E. ampicillin and discharge

A

An early sign and symptom of shock is: A. Tachycardia B. Bradycarda C. Decreased respiratory rate D. Cyanosis E. Hypotension

A

The four classic types of shock include all of the following EXCEPT: A. Traumatic B. Hypovolemic C. Obstructive D. Cardiogenic E. Distributive

A

The pathophysiology of cardiogenic shock is: A. Cardiac pump failure B. Vasodilation C. Endotoxins D. Hypoxia E. Hypovolemia

A

A 44 year old woman presents complaining of the acute onset of left eye pain while walking into a movie theater one hour ago. On physical examination, she has stable vital signs. Her visual acuity is 20/25 on the right eye and 20/200 on the left eye. Her pupillary exam is notable for a minimally reactive dilated left pupil and a steamy cornea. The left eye is red and obviously painful. Which of the following conditions is most likely? Answers: 1.acute angle closure glaucoma 2.central retinal artery occlusion 3.hypopyon 4.optic neuritis 5.vitreous hemorrhage

AACG

A 22 year old male with IVDA presents to the emergency department with a fever and dyspnea. His physical exam reveals T 101.5, clear lungs and a murmur. Of the following, the correct treatment plan is? Answers: 1.blood cultures and discharge 2.amoxicillin and discharge 3.blood cultures and admission 4.blood cultures, vancomycin, and admission 5.surgery

BC, Vanc, Admit; infective endocarditis, staph aureus

While temperatures vary with time of day and method assessed, the generally accepted upper limit of normal temperature is: A. 98.6 F (37 C) B. 99.5 F (37.5 C) C. 100.4 F (38 C) D. 101.3 F (38.5 C) E. 102.2 F (39 C)

C

All patients with shock should receive as the first priority: A. Packed red blood cells B. Trendelenburg positioning C. Supplemental oxygen D. Intravenous fluids E. Antibiotics

C

During hypovolemic shock, hypotension tends to develop after the loss of what percent of blood volume? A. 10% B. 20% C. 30% D. 40% E. 50%

C

A 10 year old boy presents with high fever and a rash. The rash started on his wrists and ankles and has spread to his trunk, palms, and soles. The correct diagnosis is most likely: Answers: 1.Rocky Mountain spotted fever 2.Lyme disease 3.scarlet fever 4.varicella 5.tinea

RMSF; rash starts wrists and ankles and spreads to palms, soles, trunk, prodrome of headache, malaise, fever; Doxycycline, even in children...chloramphenical for pregnant

Laboratory abnormalities typically seen with adrenal insufficiency include all of the following EXCEPT: A. hypokalemia B. hyponatremia C. hypercalcemia D. azotemia E. hypoglycemia

The answer is A. Hyperkalemia is seen in approximately 64% of patients with adrenal failure. Typically this is because of aldosterone production failure that normally enhances potassium excretion. Even more common is hyponatremia, present in 88% of patients. Hypoglycemia is present in two-thirds of patients and is a significant cause of morbidity and mortality associated with adrenal failure. Hypercalcemia is seen in 6 to 33% for unclear reasons; azotemia and increased hematocrit from hypovolemia may also be present.

A pattern of diffuse PR depression suggests PE A. Tachycardia is the most common finding of EKG associated with PE. B. An S1Q3T3 pattern is pathognomonic for PE. C. Right bundle branch block indicates pulmonary infarction from PE. D. A pattern of diffuse PR depression suggests PE

The answer is A. Tachycardia and non-specific ST-segment and T-wave abnormalities are the most common findings on EKG of patients with PE; however up to 25% will have EKGs unchanged from their baselines. An S1-Q3-T3 pattern is suggestive but not diagnostic of PE. Right bundle branch block may also be seen in PE, but does not indicate pulmonary infarction or severity of PE. Diffuse PR depression is typically seen in pericarditis.

Of the following choices, which diagnosis is most likely in a 53-year old cancer patient with chest pain, dyspnea, and the EKG shown in the Figure? [image] A. pulmonary embolism B. third degree heart block C. pericarditis from metastatic disease D. infectious myocarditis E. anterolateral myocardial ischemia

The answer is A. The EKG's S1Q3T3 pattern (S in lead I, Q and inverted T in lead III), while not pathognomonic, is suggestive of PE as a cause of this patient's symptoms.

All of the following are commonly used in the supportive treatment of thyroid storm EXCEPT: A. acetaminophen to manage hyperpyrexia B. amiodarone to control dysrhythmias C. corticosteriods D. oxygen E. diuretics to treat congestive heart failure

The answer is B. Amiodarone is an iodine-rich antidysrhythmic with poorly-defined effects on thyroid function that has been associated with both hyperthyroidism and hypothyroidism. It should therefore be avoided in the management of thyroid disease. Propranolol is standard therapy in thyroid storm and, in addition to its effects of adrenergic blockade, also may reduce dysrhythmias. Of note, aspirin should be avoided in the treatment of hyperpyrexia as it may increase the level of active thyroid hormone by displacing thyroid hormone from thyroglobulin.

As a general rule, when is blood transfusion indicated in the treatment of hypovolemic shock resulting from acute hemorrhage? A. first line treatment B. massive hemorrhage > 30% C. minor hemorrhage <10% D. after 1L of crystalloid bolus E. after dopamine

The answer is B. Blood transfusion can play a vital role in the treatment of hypovolemic shock from acute hemorrhage. It is generally not the first line treatment. It is indicated in massive blood loss or shock that is not responsive to significant crystalloid infusion (2L or 30 ml/kg). Pressors are not indicated in hypovolemic shock. Elderly patients and those with co-morbid illnesses may require blood products earlier than healthy adults.

Which of the following is NOT a potential indication for hyperbaric oxygen therapy? A. carbon monoxide poisoning B. decompression sickness C. necrotizing enterocolitis D. necrotizing fasciitis

The answer is C. According to the Undersea and Hyperbaric Medical Society, items A-D are all reasonable indications for the use of hyperbaric oxygen therapy. Other potential indications include patients with air or gas embolism, crush injury, and compartment syndrome.

Which of the following is a risk factor for developing pulmonary embolism A. Albumin deficiency B. Alcohol use C. Immoblization D. Corticosteroid use

The answer is C. Alcohol use has not been associated with an increased risk for pulmonary embolism. Virchow's triad of venous stasis, endothelial injury and hypercoagulability is predictive of the long list of risk factors for pulmonary embolism. Other risk factors include history of deep venous thrombosis or pulmonary embolism, malignancy, recent surgery, immobilization, hematologic abnormalities such as factor V Leiden deficiency, protein C or S deficiency, antithrombin III deficiency or homocysteinuria, among others.

A 7 year old boy presents with erythema and edema of the left eyelid. Which of the following physical examination findings is consistent with a pre-septal (or periorbital) cellulitis? Answers: 1.extraocular movement impairment 2.fever 3.pain with extraocular movement 4.proptosis 5.visual acuity impairment

fever (2) preseptal: Staph A, Strep pneum, fever, eyelid swelling, eyelid erythema, tx TMP-SMX OR Clinda PLUS amox or cefdinir....Orbital (postseptal) Cellulitis- pain, movement impairment, opthalmoplegia, proptosis, strep pneumonia, spread from sinus infection, tx with vanc and CTX

19 year old man is brought in to the emergency department by EMS after being found obtunded in his apartment by a friend. No additional history is available. On arrival, the patient is minimally responsive with sonorous respirations and a palpable rapid pulse. The most appropriate initial diagnostic test would be Answers: 1.Arterial blood gas 2.Electrocardiogram 3.Fingerstick glucose 4.Urine drug screen

fingerstick glucose

A 20 year old college student is brought to the emergency department by campus police after he was found by his roommate saying people in the TV were trying to kill him. Which of the following criteria is not an indication for admission? Answers: 1.first-time psychiatric episode 2.demonstrates risk for suicide 3.inadequate psychosocial support 4.lacks capacity to cooperate with treatment

first time

The Figure below depicts laryngoscopy and endotracheal intubation (ETI) occurring in an in-flight EMS helicopter. Regarding the patient depicted, and prehospital airway management in general, which of the following is true? Answers: 1.ETI in the helicopter cabin is technically no more difficult than it would be in the hospital emergency department. 2.For the patient in the Figure, post-intubation breath sounds will be a critical component of tube placement confirmation. 3.Postponing ETI until the aircraft is en route to the receiving center should save time when a flight crew decides a community hospital patient will require the procedure. 4.If the patient in the Figure has an easy ETI with minimal requirement for manual (bag-valve-mask) ventilation, gastric decompression (e.g. with an orogastric tube) is unnecessary. 5.Flight crew ETI success rates tend to be high in part because of their enhanced drug formulary (e.g. neuromuscular blockade) as compared to most ground EMS units.

flight crew ETI high due to advanced pharmacotherapy options

The primary etiology of peptic ulcer disease is: Answers: 1.Helicobactor pylori 2.cigarette smoking 3.NSAID use 4.stress 5.Zollinger-Ellison Syndrome

h pylori

A 42 year old former custodial worker presents stating, "I think that people can hear what I am thinking." In the emergency department, he becomes extremely agitated and threatening, and his psychoses become more severe. In choosing an antipsychotic medication, which of the following would be the most appropriate choice? Answers: 1.haloperidol 2.chlorpromazine 3.thioridazine 4.ketamine

haloperidol

Which medication is ideal for the agitated or combative patient? Answers: 1.Nitrous oxide 2.Hydromorphone 3.Haloperidol 4.Propofol

haloperidol

All of the following are true regarding acute hepatitis EXCEPT: Answers: 1.An IgM antibody to HAV indicates acute infection with HAV. 2.HBe antigen indicates active acute or chronic infection with HBV of high infectivity. 3.HBV core antibody indicates immunity to HBV. 4.Fecal excretion of hepatitis A virus usually occurs prior to symptoms of acute HAV infection. 5.The incubation period of HCV is 30-90 days.

hbv core

A 25 year old man returns to the ED, 24 hours after being released from the hospital with a new diagnosis of schizophrenia. He has recently started to take haloperidal for his psychotic symptoms. In the ED he is noted to have involuntary contractions of the muscles of the face, a protruding tongue, deviation of the head to one side, and sustained upward deviation of the eyes. Vital signs are stable, and initial labs show no electrolyte or hematological abnormalities. Of the following choices, the preferred medication for this condition is: Answers: 1.diphenhydramine 2.lorazepam 3.phenobarbital 4.metoprolol

lorazepam

With regard to U.S. Emergency Medical Services (EMS) systems, all of the following are true EXCEPT: Answers: 1.EMS systems operated by a government agency (e.g. a city department of public health) have no malpractice liability. 2.EMS is an integral component of disaster management. 3.There is evidence that helicopter EMS transport for injured patients results in improved mortality. 4.EMS medical directors do not have to be trained, or board-certified, in emergency medicine. 5.EMS fellowships are available for both ground and air transport, however no EMS subspecialty certification exists.

no malpractice

Which of the following statements regarding use of Helicopter Emergency Medical Services (HEMS) for trauma scene transports is true? Answers: 1.If the patient is in cardiopulmonary arrest, HEMS transport should be expedited to maximize chances of patient survival. 2.The flight physician should take command of the trauma scene upon HEMS arrival. 3.The potential benefit of HEMS in improving trauma outcome remains unstudied. 4.Nonphysician crews staff most HEMS vehicles in the United States 5.Transport of a patient from a motor vehicle collision to a trauma center is termed "secondary" transport if a helicopter is used.

non-physicians crews

A 2 year old is brought in to the emergency department by his mother for difficulty breathing. The mother thinks he might have swallowed or aspirated something. In regards to this patient, which of the following is TRUE regarding foreign body aspiration or ingestion? Answers: 1.A. Large objects in the upper airway typically present with mild symptoms 2.B. Objects lodged in the proximal airway have the worst prognosis 3.C. Small objects in the lower airways typically present with the most severe symptoms 4.D. The most difficult objects to remove are stiff, non-conformable objects

objects lodged in proximal airway have the worst prognosis

You suspect that your patient has swallowed a nail. Which of the following is an indication for endoscopic or surgical removal of this object? Answers: 1.A. Abdominal CT scan shows a 1cm nail in the distal sigmoid colon 2.B. Plain films do not reveal a radiopaque foreign body in the chest or abdomen 3.C. Radiography visualizes the nail in the gastric fundus 4.D. The object has progressed from the jejunum through the ileum after 24 hours

radiography visualization in gastric fundus

A 19 year old woman is brought to the emergency department by her friends because she has been saying that she is a superhero and trying to run into traffic to prove that she is indestructible. The friends report that she has been using drugs but they do not know which ones. Which of the following pairs of ocular finding and recreational drug is commonly observed? Answers: 1.dilated pupils - heroin 2.internuclear ophthalmoplegia - marijuana 3.pinpoint pupils - amphetamines 4.vertical nystagmus - phencyclidine

vertical nystagmus PCP

A 54 year old female presents with palpitations. She is otherwise asymptomatic. EKG shows atrial fibrillation. Vital signs are HR 130-150, BP 148/78, RR 16, T 36.7. What management intervention is most important to accomplish next? Answers: 1.Anticoagulation 2.Cardioversion 3.Pharmacologic ventricular rate control 4.Radiofrequency ablation

3

A trauma patient resuscitated in the ED, has a post-tube thoracostomy computed tomography (CT) scan as depicted in the figure. What finding is present? Answers: 1.Persistent pneumothorax 2.Chest tube not within the thoracic cavity 3.Pericardial tamponade 4.Aortic rupture leading to a right hemothorax

1

An 8 year old female presents with a regular, narrow-complex SVT. You diagnose AV nodal reentrant tachycardia. Which pharmacologic agent would be most appropriate for initial management? Answers: 1.Adenosine 2.Digoxin 3.Diltiazem 4.Lidocaine

1

With which of the following substances is acute withdrawal most likely life threatening? Answers: 1. Cocaine 2. Ethanol 3. Heroin 4. Lithium

2. Ethanol

A 22 year old female presents to the emergency department with a "funny feeling" in her chest. She has had similar episodes but never lasting as long as the current episode (3-hour duration). Her heart rate is 200, blood pressure is 128/68, respiratory rate is 20 and her pulse oximetry is 96%. Her EKG is shown in the Figure. The best treatment option for this patient is: Answers: 1.adenosine 2.lidocaine 3.cardioversion 4.verpamil

1

A 29 year old woman is found seizing by her husband and is rushed to the emergency department. On presentation, she is noted to have a BP of 162/112, is still seizing, and looks puffy all over. Her husband tells you that they are expecting their first child in a few months. Which of the following is the next best step in this patient's care? Answers: 1.Control the seizures with magnesium sulfate. 2.Draw blood to check CBC, LFT's, BUN, and creatinine. 3.Notify the labor floor that the patient is in the emergency department. 4.Perform a CT scan of head if seizures persist. 5.Start hydralazine to decrease the patient's blood pressure.

1

A 75 year old female is brought the to emergency department by a family member with a history of progressive forgetfulness and confusion. She has a history of dementia. The most common cause of dementia in the elderly patient is: Answers: 1.Alzheimer's disease 2.Parkinson's disease 3.Pick's disease 4.Vascular dementia

1

A blunt trauma patient presents, transported by EMS from a motor vehicle collision, with inability to provide a history, due to alcohol intoxication. He has no signs of trauma on external evaluation, but he is hypotensive. An ED ultrasound is performed at the bedside, and is depicted in the figure. Given the patients' clinical condition and image seen, what is the most likely diagnosis? Answers: 1.Free intraabdominal fluid 2.Fat embolus from femur fracture 3.Rupturing abdominal aortic aneurysm 4.Ruptured gallbladder

1

For which of the following cases is activated charcoal therapy most appropriate? Answers: 1. Acetaminophen overdose 2. Drain cleaner ingestion 3. Iron supplement overdose 4. Lithium overdose

1. Acetaminophen Overdose

A patient presents after an unknown ingestion. Her initial electrocardiogram (EKG) is shown in the image. Based on the EKG, an overdose with which of the following medications would be most likely? (EKG DEMONSTRATES QRS PROLONGATION; QRS>100) 1.Amitriptyline 2.Clonidine 3.Ibuprofen 4.Nifedipine

1. Amitriptyline

A teenager presents one hour after ingesting a "handful" of acetaminophen tablets. Which of the following statements is TRUE? 1.An acetaminophen level drawn at hour four dictates need for antidotal therapy. 2.Serial liver function tests are indicated in all acetaminophen ingestions. 3.Renal sequelae are expected. 4.The intravenous formulation of N-acetylcysteine is safer than oral N-acetylcysteine

1. An acetaminophen level drawn at hour four dictates need for antidotal therapy.

A 2 year old child presents with an overdose of her mother's iron containing multivitamins. What antidote should you consider for iron toxicity? 1.Deferoxamine 2.Glucagon 3.Methylene blue 4.Pyridoxine

1. Deferoxamine; Deferoxamine binds directly to free iron and thus is the antidote for iron toxicity. It is given intramuscularly or intravenously and often causes the patient's urine to turn color (vin rosé urine). Methylene blue is an antidote for methemoglobinemia. N-acetylcysteine is the antidote for acetaminophen. Pyridoxine is the antidote for isoniazid toxicity and glucagon can serve as an antidote for beta blocker, calcium channel blocker, or insulin overdoses.

A 41 year old man is seen in the emergency department after a street fight where he punched another man in the mouth. He has a small, jagged laceration over the dorsum of the metacarpophalangeal joint of his right hand. The wound is irrigated copiously, tetanus is given, and the wound is left open to heal by secondary intention because of the infection risk. In addition to treating Streptococcus and Staphylococcus species, antibiotics must also treat which other bacterial species? Answers: 1.Pasteurella multocida 2.Eikenella corrodens 3.Escherichia. coli 4.Actinomyces israelii

2

A 70 year old male presents with lightheadedness. He is noted to be bradycardic. His EKG below reveals: Answers: 1.first degree AV block 2.second degree AV block Mobitz Type I 3.second degree AV block Mobitz TypeII 4.complete heart block

2

A previously healthy 25 year old female arrives at the emergency department with 3 days of headache, nausea, palpitations, and diaphoresis. She initially presented 2 days ago to the hospital's walk-in clinic, where her blood pressure was found to be moderately elevated. At her clinic visit, the initial evaluation for end-organ damage was negative. In the emergency department, assessment of tests sent from the clinic visit is noteworthy for a normal TSH, normal head CT scan, and markedly elevated urine metanephrine levels. If this woman were to require emergency department therapy for hypertension, which of the following agents should be AVOIDED: Answers: 1.alpha-adrenergic receptor blocker 2.beta-adrenergic receptor blocker 3.calcium channel blocker 4.ACE-inhibitor 5.nitroprusside

2

An 18 year old hockey player is hit in the mouth with a puck, fracturing a maxillary canine tooth. He brings the severed piece of tooth with him. On physical exam, the tooth is fractured halfway between the tip and the gumline. The root of the tooth is still firmly intact. The exposed fracture site has a yellowish tinge without blood. Of the following choices, which is the most appropriate management for this patient? Answers: 1.Replace fractured piece and place acrylic splint 2.Immediate dental consult to avoid abscess formation 3.Application of calcium hydroxide, placement of aluminum foil, and dental follow-up 4.Placement of tooth fragment in saline gauze, outpatient dental follow-up 5.No specific treatment required

2

In a patient with a suspected ruptured globe from penetrating trauma to the eye, all of the following should be performed EXCEPT: Answers: 1.administration of broad spectrum antibiotic therapy 2.ascertainment of intraocular pressure via tonometry 3.ascertainment of tetanus status 4.ophthalmology consultation 5.visual acuity assessment

2

A mother brings in her 4 year old child who was happily eating "blackberries" from weeds in the garden and is now acting strangely. She has identified them as Belladonna from a quick internet search. Which physical examination finding might you also expect to find in this child? 1. Diaphoresis 2. Flushed skin 3. Miosis 4. Urinary incontinence

2. Flushed Skin, anticholinergics (hot as a hare, dry as a bone, mad as a hatter, red as a beet, blind as a bat)

A 17 year-old has presented after taking a large amount of nortriptyline prescribed for migraine prophylaxis. Clinically, you take care of stabilizing her and initiate appropriate treatment. After reviewing reference materials you calculate that she has taken a potentially lethal dose of this tricyclic antidepressant. Which of the following would you expect to see on her electrocardiogram? 1.Compacted QT intervals 2.Prolonged QRS intervals 3.Prolonged PR intervals 4.Right bundle branch block

2. Prolonged QRS intervals

Parents bring in their 13 year old girl two hours after she ingested a large amount of Acetaminophen in suicide attempt. She tearfully refuses to drink the activated charcoal. Which of the following is TRUE regarding your ability to administer the charcoal? 1. A court injunction is needed to force her to drink the charcoal 2. A nasogastric tube may be placed to facilitate treatment 3. You cannot force her to take the charcoal 4. You must get parental permission prior to treating her

2. A nastogastric tube mat be place to facilitate treatment

A 23 year old woman is dropped off by her boyfriend after an unknown overdose. You notice that she is has very large pupils and is sweating profusely. Her respiratory rate, blood pressure and heart rate are elevated. Which of the following is the most likely agent to have caused her symptoms? 1. Jimson weed 2. Cocaine 3. Heroin 4. Insulin

2. Cocaine; cocaine has sweating, jimson weed would have anhidrosis (hot as a hare, dry as a bone, mad as a hatter, red as a beet, blind as a bat)

An 84 year-old with a history of congestive heart failure is brought in by his family for vomiting and diarrhea. He also complains that things "have weird colors". He has been having odd palpitations but cannot describe them further. His family expresses their concern that he has not been taking his medications correctly. Given his presenting symptoms, which medication are you most concerned about? 1.Amiodarone 2.Digoxin 3.Diphenhydramine 4.Metoprolol

2. Digoxin; Digoxin toxicity classically presents as weakness, fatigue,nausea/vomiting/diarrhea, confusion, and a visual disturbance hallmarked byyellow/green halos around objects.

A 3 year old girl presents after accidentally ingesting an alkali drain cleaner. Which of the following statements regarding her management is true? 1.Activated charcoal should be administered 2.Endoscopy is useful in the assessment of injury 3.Gastric lavage should be performed immediately to reduce gastric injury 4.Neutralization therapy using a strong acid is warranted

2. Endoscopy is useful in the assessment of injury

A 22 year old running back is struck from behind by a 300-pound lineman. The blow occurs below the knee as his foot is firmly planted and two other linemen are holding his upper body. He presents to the emergency department with gross anterior dislocation of the tibia on the femur. His foot is cool and pale, and dorsalis pedis and posterior tibial pulses are not detected by Doppler ultrasound. What is the most appropriate management for this patient? Answers: 1.Immediate orthopedic consultation without attempts to manipulate the knee 2.Immediate arteriography to assess for popliteal artery disruption 3.Immediate reduction in emergency department under conscious sedation without X-rays 4.Open reduction in OR with exploration of popliteal artery

3

A 24 year old woman is playing racquetball and sustains a direct blow from the ball to the right eye. She presents to the emergency department complaining of eye pain and double vision. On exam, her right eye does not track properly with upward gaze. This finding suggests which of the following injuries? Answers: 1.Inferior orbital rim fracture 2.Superior orbital rim fracture 3.Inferior orbital wall fracture 4.Ethmoid fracture 5.Zygomatic arch fracture

3

A 36 year old man is a restrained driver involved in a high speed MVA where his car is struck on the driver's side door with significant intrusion. His physical exam is significant for a large contusion on his left flank. His abdominal exam is benign and rectal exam reveals a normal prostate. A Foley catheter is placed with return of gross hematuria. Which test is indicated to evaluate for the presence of urologic injury? Answers: 1.CT abdomen / pelvis with IV contrast alone 2.Ultrasound of the bladder 3.CT abdomen / pelvis with IV and transurethral contrast 4.CT abdomen / pelvis without contrast 5.Ultrasound of the kidneys

3

A 46 year old man is brought in by EMS after a motor vehicle collision in which he was an unrestrained driver. Although he has no obvious injury to his head or neck, he complains of chest pain and appears very short of breath. His vital signs are: T 99.2 F, BP 85/57, HR 123, RR 36, SpO2 95% on non-rebreather. The CXR demonstrates a tension pneumothorax. Of the following, which is the most appropriate next step in this man's care? Answers: 1.Performance of a chest CT scan to further delineate the pathology 2.Placement of a chest tube followed by a chest xray to determine proper placement 3.Placement of a needle decompression device, followed by repeat CXR 4.Transfusion of 2 units of O-negative packed red blood cells

3

A 5 year old boy presents with fever and sore throat. On physical exam, the child has enlarged tonsils with exudates. Which of the following is true? Answers: 1.The etiology of the infection is most likely bacterial 2.Viruses rarely cause exudative pharyngitis 3.If the infection is bacterial, the primary role of antibiotic therapy is to prevent complications 4.Treatment of choice is azithromycin 5.Vaccination prevents recurrence

3

A 25 year old presents with an ingestion of acetaminophen 2 hours prior to arrival. Which of the following statements is TRUE? 1.AST of 32 and ALT of 27 from arrival labs indicate the absence of hepatotoxicity from this ingestion. 2.An acetaminophen level of 84 mg/dl from arrival labs necessitates use of n-acetylcysteine 3.Activated charcoal is indicated to treat this ingestion 4.Acetaminophen toxicity is predicted to occur at a dose of 20 mg/kg.

3. Activated charcoal is indicated to treat this ingestion; NAPQI -- the prime toxic mediator -- builds up when glutathione stores deplete and thus causes hepatotoxicity. The first stage of acetaminophen toxicity is largely asymptomatic. The toxic acetaminophen dose, when a single ingestion of nonsustained-release preparation is taken, is about 140 mg/kg. Therapy is guided by the Rumack-Matthew nomogram, provided the ingestion is an acute one involving nonsustained-release preparations. The antidote, N-acetylcysteine, prevents toxicity by inhibiting the binding of NAPQI to hepatocytes.

A 72 year-old presents with an intentional overdose of a bottle of aspirin about 3 hours prior to presentation in the ED. Which of the following arterial blood gas results would you expect to come from this patient? 1.pH 7.14 pCO2 68 pO2 102 HCO3 23 2.pH 7.33 pCO2 48 pO2 58 HCO3 29 3.pH 7.45 pCO2 21 pO2 124 HCO3 14 4.pH 7.47 pCO2 31 pO2 96 HCO3 25

3. pH 7.45 pCO2 21 pO2 124 HCO3 14; metabolic acidosis with respiratory alkalosis

A 23 year old man is stabbed in the anterior neck with a 3-inch knife during a street fight. At the scene, there is some bleeding, which is controlled with direct pressure. He presents to the emergency department breathing comfortably and in no distress. His pulse is 88, blood pressure 126/76, and oxygen saturation 99% on room air. There is a 1cm laceration 2cm above the right sternoclavicular junction, lateral to the trachea. There is mild oozing and no obvious underlying hematoma. There is no obvious subcutaneous air, and he has clear lung sounds. What is the most appropriate management for this patient? Answers: 1.Immediate operative exploration 2.Local wound exploration and discharge home if no significant injury identified 3.Local wound exploration and discharge home after 6-hour observation period 4.Angiography, esophogram, and admission for observation 5.CT scan of the neck and discharge home after 6 hours of observation

4

A 23-year-old male presents after a syncopal episode. EKG findings include normal sinus rhythm, a short PR interval (less than 0.12 seconds), QRS duration of 0.11seconds, and the presence of a "delta wave" (a slurred upstroke to the QRS complex). What condition most likely caused the syncopal episode? Answers: 1.Brugada syndrome 2.Dextrocardia 3.Vasovagal reaction 4.Wolff-Parkinson-White syndrome

4

A 32 year old man is struck several times in the head with a baseball bat. Upon emergency medical service arrival, he is mildly confused, vomits once, and complains of a severe headache. The emergency medical technicians establish two large-bore IVs. Prior to arrival at the emergency department, he loses consciousness and begins to seize. He is actively seizing when he is brought into the trauma bay. What should be the first step in the management of this patient? Answers: 1.Administration of 2 liters NS bolus 2.Administration of phenytoin 1000mg IV 3.Administration of mannitol 50 g IV 4.Rapid sequence intubation using paralytic agent 5.Emergency craniotomy

4

A 40 year old male presents to the emergency department complaining of severe ankle pain after inverting the foot during a soccer game. The triage nurse records the following vital signs: temperature 98.8, pulse 94, respiratory rate 18, BP 188/118. Which of the interventions below is the most appropriate step to take in response to the blood pressure assessment? Answers: 1.Administer a sublingual antihypertensive agent since the patient probably only has an ankle sprain and will not need an intravenous line 2.Establish intravenous access in order to optimize the onset of action of parenteral antihypertensive medications 3.Ignore the blood pressure since the patient is asymptomatic other than having ankle pain 4.Order an antihypertensive agent to be given in the emergency department because the patient will be discharged with a prescription for one

4

A 46 year old construction worker falls 6 feet off a ladder onto a concrete surface and has sudden and severe low back pain. The pain radiates down his right leg and he develops numbness over the anterior shin and dorsum of the foot. On physical exam he has decreased sensation to pinprick over the dorsum of the right foot (medially) and some weakness in right foot dorsiflexion. At which level is a protruding intervertebral disc most likely? Answers: 1.L1-L2 2.L2-L3 3.L3-L4 4.L4-L5 5.L5-S1

4

A 47 year old man with a history of alcohol abuse presents to the emergency department after having a seizure. His past includes both seizures and blackouts. His last alcoholic drink was the previous evening. This morning he experienced palpitations, diaphoresis, and dizziness before losing consciousness and having a seizure lasting under a minute. Which of the following is a true statement with regard to alcohol and its association with seizures? Answers: 1.In people with an underlying seizure disorder, excessive alcohol intake is a risk-factor for seizure due to increased likelihood of head injury, predisposition to metabolic disorders, and lowered seizure threshold. 2.Alcohol intake itself can precipitate seizures due to the neurotoxic effects of alcohol and it metabolites 3.Cessation of alcohol can precipitate seizures as part of the alcohol withdrawal syndrome 4.All of the above statements are true. 5.All of the above statements are false.

4

A 60 year old woman presents to the emergency department with palpitations. Her EKG, shown in the Figure, reveals: Answers: 1.normal sinus rhythm 2.atrial flutter 3.ventricular tachycardia 4.atrial fibrillation

4

A 76 year old restrained driver is involved in a head-on collision at about 35 mph. He arrives at the emergency department in a cervical collar and on a backboard. His only complaint is neck pain, and he has mild posterior neck tenderness. A CT scan of the neck shows no fracture and only degenerative arthritis. Upon re-evaluation you note the patient has difficulty raising his arms against gravity and there is decreased grip strength bilaterally. The remainder of his neurological exam is normal. What is the most appropriate management for this patient? Answers: 1.Immediate neurosurgical decompression 2.Flexion and extention radiographs to rule out ligamentous injury 3.Discharge home with a hard cervical collar with neurosurgical follow-up 4.Administration of IV steroids and ordering of cervical MRI 5.Reassurance and discharge with NSAIDs given the non-anatomical distribution of weakness

4

A patient falls onto his face, and has a CT scan of the face as shown in the Figure. Which indirect finding suggestive of possible facial fracture is present on the CT? Answers: 1.Exopthalmos 2.Extra-sinus air 3.Nasal fracture 4.Fluid (blood) in the sinuses

4

A patient with a ventriculo-peritoneal shunt presents to the E.D. with lethargy and vomiting. A CT scan obtained emergently (see the Right image in the Figure) is compared with a CT scan (see the Left image in the Figure) from a month ago. What is the diagnosis? Answers: 1.encephalitis 2.meningitis 3.pseudotumor cerebri 4.hydrocephalus and shunt malfunction 5.subarachnoid hemorrhage

4

A right-handed patient sustains a circumferential burn (see figure) to the distal right forearm and hand, which is cool despite warm ambient temperature. Regarding the initial assessment and management of the patient, which of the following is correct? Answers: 1.Prophylactic penicillin should be given to any patient with a significant burn 2.Pain medication should be withheld pending obtaining operative consent for emergency burn debridement 3.As shown in the Figure, the burn represents about 10% of total body surface area 4.The involved areas of this 3rd-degree burn would be expected to be mostly insensate

4

A young male patient presented to the emergency department with a history of single gunshot wound from unknown source and caliber. He was hemodynamically stable and had little pain in the right lower quadrant of the abdomen, which was the site of two wounds about 6 inches apart (see Figure). No other wounds were identified during physical examination. Which of the following statements regarding this case is true? Answers: 1.If it is suspected that the two wounds are from the same missile, the emergency department physician's documentation should note which wound is the entrance, and which is the exit 2.The two wounds are more likely than not the result of two separate missiles 3.The superior-medial wound (at the top right of the Figure) is likely the entrance wound, and the inferior-lateral wound is likely the exit wound, of a single missile 4.Wound description is essential for the emergency medicine specialist. However, description of a wound as to entrance or exit is best left to forensic examination.

4

An 82 year old woman with osteoporosis slips and falls onto her right hip. She cannot get up and is brought to the emergency department by ambulance. As you enter the room you notice her right leg is abducted and externally rotated. What type of injury does she most likely have? Answers: 1.Posterior hip dislocation 2.Subtrochanteric femur fracture 3.Intertrochanteric femur fracture 4.Femoral neck fracture 5.Acetabular fracture

4

During a bar fight, a 42 year old man is stabbed in the left side with an unknown weapon. He presents to the emergency department with dyspnea, pulse of 108, blood pressure of 138/92, and oxygen saturation of 94% on room air. He has absent breath sounds on the left side; you note a small puncture wound in the midaxillary line at the level of the 10th rib. His abdominal exam is normal. Two large-bore IVs are established. What is the appropriate management of this patient? Answers: 1.Endotracheal intubation, left-sided chest tube, portable chest x-ray, and admission 2.Left-sided chest tube, portable chest x-ray, and admission 3.Left-sided chest tube, portable chest x-ray, diagnostic peritoneal lavage, and admission 4.Left-sided chest tube, portable chest x-ray, and abdominal CT scan 5.Endotracheal intubation, portable chest x-ray, exploratory laparotomy in the OR, and admission

4

Following a motor vehicle crash, a 25 year old man presents complaining of a painful right eye. Visual acuity is 20/200 in the right eye and 20/25 in the left eye. The right eye protrudes from the orbit and the patient has right eye pain with extraocular movement. What is the most likely cause of his symptoms? Answers: 1.chemosis 2.hyphema 3.orbital blow-out fracture 4.retrobulbar hematoma 5.ruptured globe

4

In a patient with malignant hypertension, the patient's blood pressure should be reduced to what value in the initial 2 hours of treatment? Answers: 1.120/80 mm Hg 2.100/70 mm Hg 3.90/60 mm Hg 4.75% of the pretreatment mean arterial pressure (MAP) 5.50% of the pretreatment MAP

4

In differentiating high voltage electrical injury from lightning injury, which of the following is your best discriminator? Answers: 1.Loss of consciousness 2.Cardiac arrest 3.Fractures or dislocations 4.Deep burns

4

Of the following choices, which diagnosis is most likely in a 35-year old female with intermittent palpitations and the EKG shown in the Figure? Answers: 1.digoxin overdose 2.asthma 3.pericarditis 4.Wolff-Parkinson-White syndrome

4

The best treatment, of the options below, for a patient with second degree AV block Mobitz Type II is: Answers: 1.amiodarone 2.epinephrine 3.aspirin 4.transvenous pacing 5.lidocaine

4

Which of the following statements regarding fever and WBC is TRUE? Answers: 1.Elevated WBC indicates serious bacterial infection. 2.WBC is sensitive for serious bacterial infection. 3.WBC is specific for serious bacterial infection. 4.WBC is a poor discriminatory predictor of serious bacterial infection. 5.WBC is of no clinical value.

4

A 42 year-old woman presents with an overdose of her Xanax (alprazolam) that her family indicates she has been taking for years to help with her anxiety. The bottle indicates that the prescription was filled yesterday with 90 pills and is now empty. The patient is minimally responsive to painful stimuli and does not react when you suction secretions out of her posterior pharynx. What is your next management step? 1.Administration of flumazenil 2.Administration of narcan 3.Close observation 4.Intubation for airway support

4. Intubation for airway support

A 55 year-old man is brought down from the outpatient procedures clinic after becoming severely short of breath during an endoscopy under light sedation. His pulse oximeter is reading 100% on a non-rebreather mask. You notice an interesting discoloration of his blood when it is drawn. What antidote should be administered? 1.Amyl nitrite 2.Deferoxamine 3.Hydroxycobalamin 4.Methylene blue

4. Methylene blue; Nitrates/nitrites, local anesthetics, dapsone, and phenazopyridine are the common causes of methemoglobinemia. Methemoglobinemia causes the oxygen dissociation curve to shift to the left, making the remaining hemoglobin less likely to give up oxygen to the tissues. Blood from patients with methemoglobinemia is a chocolate brown color. Methylene blue is the antidote. Pulse oximetry is unreliable in patients with methemoglobinemia, since the pulse oximeter cannot differentiate oxyhemoglobin from methemoglobinemia.

The clinical presentation of clonidine toxicity most closely mimics toxicity from which of the following classes of medication? 1.Beta blockers 2.Cholinergics 3.Stimulants 4. Opioids

4. Opioids; the hallmark signs and symptoms of clonidine toxicity include:hypotension, bradycardia, mental status change, respiratory depression, and miosis. The presentation very closely mimics opioid toxicity.

For which of the following cases (all of which are characterized by an ingestion history known with certainty) is gastric lavage most likely indicated? 1. Adult patient, ibuprofen ingestion (800 mg/tab x 5 tabs, 20 minutes prior to arrival (PTA) in the ED 2. Adult patient, kerosene ingestion (8 oz, 20 minutes PTA) 3.Adult patient, metoprolol ingestion (100 mg/tab x 100 tablets, 6 hours PTA) 4.Adult patient, nortriptyline ingestion (50 mg/tab x 100 tablets, 45 minutes PTA)

4.Adult patient, nortriptyline ingestion (50 mg/tab x 100 tablets, 45 minutes PTA)

A 22 year old man is punched in the nose during a fight. He presents to the emergency department with obvious nasal bone deformity. Pressure controls the bleeding. Physical exam reveals no maxillary bone or orbital rim tenderness, intact vision and extraocular movement. The oropharynx and mandible are unremarkable. Nasal inspection reveals a swollen, ecchymotic, tender nasal septum. Which of the following is the most appropriate initial step? Answers: 1.Plastic surgery consult for immediate reduction of nasal fracture 2.Facial CT scan to rule out more serious facial fractures 3.Outpatient follow-up with an ENT specialist to surgically correct a deviated septum 4.Needle aspiration of the septal hematoma 5.Incision and drainage of the septal hematoma followed by nasal packing

5

A 40 year old female presents to the emergency department complaining of a few days of headaches, excessive sweating, anorexia, heat intolerance and palpitations. She has also been having upper respiratory symptoms over the past week. She is found to have a blood pressure of 170/106 and an EKG, urinalysis, fundoscopic examination, serum creatinine, and neurological evaluation are negative. What is the next step in the evaluation/management? Answers: 1.Administer sublingual nifedipine while the work-up continues 2.Perform CT scan of the abdomen 3.Avoid sublingual or intravenous therapy in the ED and prescribe an oral beta-blocker 4.Schedule a clonidine suppression test to evaluate for pheochromocytoma 5.Obtain a medication history

5

All of the following are common causes of septic shock EXCEPT: A. Streptococcus pneumoniae B. Escherichia coli C. Staphylococcus aureus D. Group A beta-hemolytic Streptococcus E. Pseudomonas aeruginosa

D

The best IV access for volume resuscitation of the hypovolemic patient is: A. intraosseous line B. PICC line C. triple-lumen internal jugular central venous catheter D. 16g catheter in the antecubital fossa E. 22g catheter in the dorsum of the hand

D

A 27 year old man is shot in the right leg. He is unconscious. The wound appears to be pulsatile. The medics report he has lost a lot of blood. His heart rate is 160, and his BP is 70/30. He has received 2 liters of IVF normal saline. The next step in management would be: A. Administer Type O Rh+ blood B. Check a hemoglobin level and hematocrit C. Wait for type-specific blood D. Wait for cross-matched blood E. Give more saline

A

A college student who had a mild upper respiratory tract infection last week, presents during the spring. He appears toxic, with fever, headache, and a rash (see figure) which was also noted on the wrists, ankles, flanks, and axilla. Of those listed below, which is the most likely diagnosis? [image] A. meningococcemia B. Lyme disease C. pemphigus vulgaris D. angioedema E. herpes zoster

A

For a patient who has suffered a severe anaphylactic reaction, which of the following medications would serve little purpose if prescribed upon discharge? Answers: 1.prednisone 60mg daily for 5 days 2.benadryl 25-50mg every 4-6 hours 3.Epi-Pen 4.albuterol inhaler 2 puffs every 4 hours

Albuterol

A 4 year old boy presents with fever, sore throat and stridor. Physical exam reveals T103 in an ill-appearing, drooling, stridorous child in mild respiratory distress. The next steps include: Answers: 1.albuterol 2.antibiotics and airway management 3.chest X-ray 4.dexamethasone 5.observation

Antibiotics and airway management; A child presenting to the emergency department with stridor most likely has croup, with epiglottitis being a less common - but more serious - etiology. Croup tends to be the etiology in younger, nontoxic-appearing children, who usually have a characteristic barking ("seal") cough. Treatment of croup includes cool mist (though the literature supporting this is limited), racemic epinephrine (which is probably no better than nebulized l-epinephrine), and steroids. Epiglottitis is a true emergency. It presents in children who are older, with high fever, and who are ill appearing. One of the key clinical features is drooling, which indicates swelling and pain interfering with handling of secretions.

A 19 year old man is stabbed in the left chest in the 3rd intercostals space just to the left of the sternum. His vital signs are: T 97.9, HR 130, BP 60/48, RR 18, SPO2 84%. He has significant JVD and his lungs are clear. The patient's diagnosis is: A. Tension pneumothorax B. Cardiac tamponade C. Lung laceration D. Spinal cord injury E. Hypovolemic shock

B

All of the following are signs and symptoms of hypovolemic shock EXCEPT: A. Tachycardia B. Warm, moist skin C. Cool, clammy skin D. Narrow pulse pressure E. Decreased capillary refill

B

As compared to adults, children with shock usually: A. Have more reliable signs and symptoms B. Are able to maintain their blood pressure better C. Have similar epidemiology (i.e. causes for shock states) D. Do not need specialized care E. Have different treatment priorities

B

What is treatment of conjunctivitis?

Bacterial- unilateral thick purulent discharge tx with topical abx (erythro, tmp-smx topical) Viral- bilateral clear watery discharge (adenovirus)

Which of the following is a common physiologic finding in septic shock? A. Normothermia B. Increased cardiac index C. Increased pulmonary wedge pressure D. Decreased urine output E. Increased systemic vascular resistance (SVR)

D

Women who develop pre-eclempsia in their first pregnancy have a long-term risk for which of the following? Answers: 1.diabetes mellitus 2.hypertension 3.liver disease 4.habitual miscarriage 5.renal failure

DM

All of the following are common complications of septic shock EXCEPT: A. Adult respiratory distress syndrome (ARDS) B. Acute tubular necrosis (ATN) C. Disseminated intravascular coagulation (DIC) D. High-output congestive heart failure (CHF) E. Pulmonary embolus (PE)

E

Shock is defined as: A. hypovolemia B. tachycardia C. hypotension D. altered mental status E. inadequate tissue and organ perfusion

E

Esophageal foreign bodies: Answers: 1.Occur at the upper Esophageal Sphincter 2.Require surgical treatment 3.Commonly perforate 4.Can be treated with endoscopy 5.Cannot be treated with medications

Endoscopy

With growing regionalization of care for many patient types and conditions, the traditional province of "prehospital" care is growing to include "out-of-hospital" care. The increasing need for critical care transport to regional centers has translated into regularly-occurring out-of-hospital, intratransport utilization of all the following EXCEPT: Answers: 1.extracorporeal membrane oxygenation (ECMO) 2.intra-aortic balloon counterpulsation 3.mechanical ventilation 4.ventilation with nitric oxide-containing gas 5.continuous propofol infusion

ECMO

Which of the following statements regarding the topical ophthalmic anesthetic proparacaine is TRUE? Answers: 1.It is an amide. 2.It is an ester. 3.It is cardiotoxic. 4.It is often prescribed for home-going analgesia in patients with corneal pathology. 5.It stains the tears orange.

Ester (2), check for allergies as some patients have ester or amide allergies, only used in ED as can decrease corneal healing

What corneal pathology is predicted in a patient with a foreign body lodged in the conjunctiva of the upper eyelid? Answers: 1.branching abrasions with terminal bulbs 2.corneal ulcer directly opposite the location of the foreign body 3.dendrites 4.rust ring 5.vertical corneal abrasions

vertical corneal abrasions

Since hiking in Connecticut 2 weeks ago, a 27-year-old woman has developed a rash and low-grade temperature. The anular-macular rash appears to be spreading from one location. She remembers multiple insect bites. The correct diagnosis is: Answers: 1.Rocky Mountain spotted fever 2.Lyme disease 3.scarlet fever 4.varicella 5.tinea

Lyme disease-erythema migrans, Doxy or Amox

A collection of purulent material in the location as depicted in the figure represents a: (underneath distal phalynx) Answers: 1.paronychia 2.herpetic whitlow 3.felon 4.furuncle 5.cellulitis

Felon; felon is infection underneath distal phalynx, if tense suspect abscess and do I&D; paronychia is infection/swelling of nail bed, herpetic whitlow will have blisters

A patient presents complaining of severe pain and swelling in the distal aspect of his index finger. He has no history of trauma. On exam he has tense swelling and redness on the pad of the digit. Of the following, which is the best next step? Answers: 1.antibiotics and discharge 2.consult orthopaedics for pinning of a probable fracture 3.incision and drainage 4.radiograph 5.treat for herpetic whitlow and discharge

I&D

Which of the following analgesics operates by a non-opioid mechanism? Answers: 1.hydromorphone 2.meperidine 3.fentanyl 4.ketorolac 5.codeine

Ketorolac

A 20 year old otherwise healthy obese woman presents complaining of headache, nausea, vomiting. She is afebrile and her vital signs are normal. Physical examination reveals papilledema, but an otherwise normal neurological exam without meningismus. Non-contrast CT head scan is negative for intracranial pathology. What is the most appropriate next step? Answers: 1.IV antibiotics 2.EEG 3.MRI/MRA 4.erythrocyte sedimentation rate/C-reactive protein 5.lumbar puncture

LP; psudomotor cerebri (increased ICP, papilledema, NL CT and CSF)

A 32 year old G1P0 at 33 weeks EGA comes into the emergency department complaining of a severe headache. She has contractions every 3 minutes. She is experiencing flashes of light in front of her eyes. Her pregnancy has been uncomplicated until this time, and her only medical problem is mild asthma. Her vital signs are: T 36.5 C (97.7 F), BP 172/114, P 78, R 14, and a room air SpO2 99%. Her lungs have bilateral crackles at the bases, and her cervix is dilated at 3 cm and effaced at 50%. Her urinalysis has 2+ protein, and her complete blood count shows: WBC 8,000/mm3, hematocrit 38%, platelets 215,000/mm3. Her BUN and creatinine are normal, her AST is 250 U/L, and her ALT is 316 U/L. The electronic fetal monitor shows a reactive and variable heart tracing at a rate in the 150s. What is the appropriate next step in management for her? Answers: 1.Start PGE2 gel 2.Discharge the patient and tell her to return for a cervical check the next day 3.Give furosemide 4.Start terbutaline, as a tocolytic that could prophylax against an asthma flare as well 5.Administer magnesium sulfate

Magnesium

All of the following may be used in the treatment of acute angle closure glaucoma (AACG) EXCEPT? Answers: 1.carbonic anhydrase inhibitors 2.laser iridectomy 3.mannitol 4.mydriatic agents 5.topical beta blockers

Mydriatic agents. In fact, mydriatic agents are contraindicated. Miotic drops, like Pilocarpine by contrast, are used. Other agents employed in the treatment of AACG include: carbonic anhydrase inhibitors (decreased aqueous production), topical beta cblockers (decreased aqueous production), mannitol (decreased intraocular pressure), and iridectomy

A 16 year old male presents with a large swollen right knee. He denies any trauma. His physical exam reveals T101 and a swollen right knee. The knee is hot, and there is pain with motion. Also noted is a diffuse rash. The correct diagnosis is: Answers: 1.gout 2.pseudogout 3.juvenile rheumatoid arthritis 4.Neisseria gonorrhea 5.Osgood-Schlatter

N. Gonorrhea

A mother brings her 35 year old son to the emergency department because of tremor and mutism for the past three days. His mother found him in his room this morning lying stiffly in his bed, soiled with urine and feces. He appears confused and will not respond to questions. He was diagnosed with schizophrenia last year and has been on several medications. Last month after his most recent hospital admission for schizophrenia, he was discharged with a prescription for haloperidol. On physical exam, he is visibly diaphoretic and has vital signs as follows: T 102.7, BP 140/98, P 112, R 12. His neuromuscular exam shows extremely rigid extremities, and his laboratory values are notable for a white blood cell count of 15000/mm3 and abnormally elevated creatine phosphokinase levels. What is the most likely explanation for these findings? Answers: 1.neuroleptic-induced acute dystonia 2.neuroleptic malignant syndrome 3.schizophrenia, catatonic type 4.tardive dyskinesia

NMS

Regarding optic neuritis, all of the following are true EXCEPT: Answers: 1.associated with afferent pupillary defect 2.commonly affects color vision 3.may be the initial manifestation of multiple sclerosis 4.oral steroids are indicated for treatment 5.preferentially affects women

Oral steroids, patient should receive IV steroids not oral

A 19 year old presents with bizarre behavior and a friend admits to use of PCP. What ocular findings would you expect? 1.Afferent pupillary defect 2.Monocular diplopia 3.Nystagmus 4.Mydriasis

Nystagmus

Which of the following women are considered at increased risk for pre-eclampsia? Answers: 1.cigarette smoker 2.multiparous 3.obese 4.over 20 years old 5.woman with a single intrauterine pregnancy

Obese

What is the most common cause of fatal anaphylaxis? Answers: 1.foods, including shellfish, nuts, and eggs 2.insect stings 3.penicillin 4.radiographic contrast dye

PCN

Which of the following drugs is NOT associated with potential toxic side effects related to the inner ear? Answers: 1.gentamicin 2.aspirin 3.furosemide 4.penicillin 5.phenytoin

PCN

You have a 3 year old female present with her mother with compaints of two days of left ear pain. On exam, you are unable to visualize the tympanic membrane due to an obstructing mass. You suspect a foreign body. Which of following is TRUE regarding the removing a foreign object from the patient's ear canal? Answers: 1.A. Avoid suction as it may lead to a perforation of the tympanic membrane 2.B. Referral to an otolaryngologist for foreign body removal under general anesthesia may be required in an uncooperative infant 3.C. To remove a live insect from the external ear canal, grasp a leg with hemostats and pull firmly 4.D. You should avoid the use of lidocaine and other topical anesthetics due to the risk of localized tissue ischemia

Referral to ENT for removal

A 22 year old college student arrives in the emergency department complaing of painful, tearing, and redness in the right eye. Blinking increases the pain. She wears contact lenses for distance vision but has no other ocular history. During your examination, you evert her upper eyelid. ED management of this patient includes: Answers: 1.A. Avoidance of Fluorescein examination due to concern for globe rupture 2.B. Immediate application of pilocarpine drops 3.C. Instructions to wear an eye patch for the next 48 hours 4.D. Removal of the foreign body with a moistened cotton swab

Removal of foreign body with cotton swab

Which of the following is the most serious toxic effect of the use of MgSO4 in treatment of eclampsia? Answers: 1.Loss of deep tendon reflexes 2.nausea and vomiting 3.neonatal hypotonia 4.renal failure 5.respiratory depression

Respiratory depression

A 2 and a half year old girl is brought in to the emergency department by her mother for "loud breathing" and a fever. The mother states the child has been previously healthy and is up to date on all her vaccines. Initial evaluation of the child reveals an ill-appearing child with her head propped upright, who is drooling and making stridorous noises. Her vital signs are notable for a temperature of 104.2 F. Which of the following is most likely in this child? Answers: 1.Chlamydia pneumonia 2.viral croup 3.Stapholococcus aureus retropharyngeal abscess 4.choanal atresia 5.viral upper respiratory tract infection

Staph aureus retropharyngeal abscess; This patient exhibits several classic symptoms of retropharyngeal abscess including fever, neck stiffness, and drooling, as well as her generally toxic appearance. Retropharyngeal abscess in children is often associated with foreign body ingestion leading to perforation of the hypopharynx or esophagus

Which pharmacologic treatment for hyperkalemia works through stabilization of cardiac membranes? A. Calcium B. Insulin and glucose C. Magnesium D. Bicarbonate

The answer is A. "Immediate antagonism of K+ at the cardiac membrane is achieved with IV administration of calcium chloride or gluconate. This is indicated in patients with unstable dysrhythmia or hypotension."

Regarding the pathogens involved in community-acquired pneumonia, which of the following is true? A. Co-infection with multiple bacteria, such as Chlamydia and Strep pneumoniae commonly occur B. Etiologic agents for patients admitted to the ICU with pneumonia most commonly include Neisseria meningitidis and Strep pneumoniae C. Milder cases of community acquired pneumonia are frequently caused by Chlamydia D. Q fever, caused by Coxiella burnetii, may present as pneumonia, particularly in patients exposed to rabbits

The answer is A. Co-infection with multiple bacteria, such as Chlamydia and S. pneumoniae, is a well-recognized occurrence and should be sought out to ensure appropriate antibiotic coverage.

With regard to laboratory findings in hypothyroidism, which of the following is false? A. Free thyroxine (T4) is always depressed in hypothyroid states. B. T3 level may be normal in hypothyroid states. C. Total thyroxine levels may be normal due to elevated thyroxine-binding globulin (TBG) levels. D. Free T4 and TSH levels are typically low in secondary and tertiary hypothyroidism. E. Serum thyroid-stimulating hormone (TSH) is the most sensitive test to diagnose primary hypothyroidism.

The answer is A. Free T4 may be normal in early stages of hypothyroidism due to physiologic compensation from elevated TSH levels.

A 36 year old male backpacking in the wilderness loses his way in a snowstorm. Temperatures are well below zero degrees and his clothing is inadequate. He is rescued 5 days later and presents to the ED. Rescue crew has already initiated passive rewarming and have removed patient's damp clothing. On arrival, vital signs show pulse of 100 and temperature of 35.5C. On physical exam, you note patient has several toes that are purple with hemorrhagic blisters on his feet. Which of the following is the most appropriate initial management? A. Immersion in warm water bath B. Debridement of necrotic tissue C. Tetanus prophylaxis D. Administration of morphine

The answer is A. Immersion of the affected extremity is the mainstay of treatment for patients with frostbite. Numbness of the affected area is the most common initial symptom and severe pain is frequently encountered after rewarming. Tetanus prophylaxis and debridement is indicated , but is not the most appropriate initial step in the management of patients with frostbite. CDEM Hypothermia.

A pneumonia caused by which of the following organisms is classically associated with currant jelly sputum? A. Klebsiella B. Mycoplasma C. Strep. pneumoniae D. viral E. H. influenzae

The answer is A. Klebsiella is not a common cause of community-acquired pneumonia. It generally occurs in the elderly, smokers, alcoholics, and those with other co-morbidities. It is classically associated with a bulging fissure on chest X-ray and currant jelly sputum.

An 82 year old man presents from the nursing home with fever, cough, nausea, vomiting and diarrhea. His vitals signs are T 102, P 65, BP 100/50, RR 24, and SpO2 92%. The most likely causative organism is: A. Legionella B. H. influenzae C. Strep. pneumoniae D. viral E. Mycoplasma

The answer is A. Legionella is an atypical pathogen commonly causing pneumonia. It is often found in the elderly or others with co-morbid illnesses. It is classically associated with GI symptoms and relative bradycardia.

In a 70kg male DKA patient with serum glucose of 573 mg/dL, all of the following statements with regard to fluid and electrolyte imbalances are true EXCEPT: A. A normal magnesium level is reassuring and obviates the need for magnesium replacement. B. Despite a serum potassium level of 4.8 mEq, the patient is probably total body potassium depleted. C. The patient is likely to be total body phosphorus depleted. D. Serum sodium of 129 mEq represents dilutional hyponatremia and the corrected value is approximately 137 mEq. E. Total body water deficit is approximately 5L.

The answer is A. Patients with DKA are typically severely dehydrated with a total body water deficit of approximately 70-80 mL/kg, in addition to being total body depleted of potassium, magnesium, and phosphorous despite initially normal serum levels of these electrolytes.

Regarding the etiology of viral pneumonia, which of the following is true? A. Influenza virus is the most common cause of viral pneumonia in children B. Cytomegalovirus does not usually cause pneumonia in immunocompetent adults C. Parainfluenza virus is the most common cause of viral pneumonia in adults D. Varicella zoster causes pneumonia more frequently in children than in adults

The answer is B. Varicella zoster virus (VZV), the etiologic agent of chicken pox, more commonly presents as pneumonia in adults, especially smokers or pregnant women. Influenza outbreaks, which occur in winter months, may cause up to 40,000 deaths annually in the United States with elderly people making up more than 90% of the victims.

Regarding the role of malignancy in the diagnosis of pulmonary embolism (PE): A. 25% of PE patients without identifiable risk factors are diagnosed with cancer within 2 years. B. Hematologic malignancies such as leukemia and have the highest incidence of venous thromboembolism C. The risk of PE is decreased in patients on chemotherapy D. Autopsy studies indicate that greater than 60% of patients who die of ovarian cancer have PE.

The answer is A. Patients with esophageal and laryngeal cancer, as well as leukemia and lymphoma have a low incidence of PE, whereas those with ovarian or colon cancers are at higher risk for developing PE. Autopsy studies indicate that up to 30% of patients who die of ovarian cancer have PE. Chemotherapy increases the risk of developing PE. Development of PE without any identifiable risk factors warrants a search for an underlying malignancy.

Regarding the treatment of hyperosmolar hypertonic nonketotic coma (HHNC) and its associated symptoms, which of the following is correct: A. Phenytoin (Dilantin) is often ineffective for seizures associated with HHNC. B. Half of the fluid deficit should be corrected over the first hour and the remainder over the following 8 hours. C. Since patients are not acidotic, close monitoring of glucose is not necessary. D. Hyperosmolarity should be corrected within the first few hours in the emergency department. E. In HHNC patients with severe dehydration, bleeding diathesis is a major clinical concern.

The answer is A. Phenytoin (Dilantin) is contraindicated in patients with HHNC as it may impair endogenous insulin release and is often ineffective in the management of seizures associated with HHNC. Half of the fluid deficit should be replaced over the first 8 hours, and the remainder over the ensuing 24 hours. Glucose must be tightly monitored as fluid resuscitation alone may normalize serum glucose or precipitate hypoglycemia in aggressive fluid resuscitation. Too-rapid correction of hyperosmolarity may result in development of cerebral edema, especially in children. Subcutaneous heparin should be considered in patients with severe dehydration due to increased risk of thrombosis from hypovolemia and hyperviscosity.

Which of the following are associated with a higher risk of spontaneous pneumothorax? A. Cigarette smoking B. Female gender C. Physical exertion D. Trisomy 21

The answer is A. Physical exertion has not been associated with an increased risk of spontaneous pneumothorax.

Regarding pulsus paradoxus and asthma, which of the following statement s is correct A. The presence of pulsus paradoxis in asthma indicates severe disease B. Pulsus paradoxus is pathognomonic for asthma C. The absence of pulsus paradoxis in asthma rules out severe disease D. Pulsus paradoxus is a fall in systolic blood pressure during inspiration

The answer is A. Pulsus paradoxus is defined as a fall in systolic blood pressure of greater than 10mm Hg upon inspiration. It is typically present during acute asthma exacerbations in severe asthma; however, its absence does not rule out severe disease. Although initially present, a pulsus paradoxus may disappear after only minimal improvement in air flow through the larger airways. Pulsus parodoxus may occur in other diseases besides asthma (for example, pericardial tamponade).

A 48 year old farmer is plowing his field when a thunderstorm rapidly overcomes him. Drivers on a nearby highway see him struck by lightening. You respond to the scene with EMS. What is the least likely finding on physical exam? A. cardiac asystole B. Glascow Coma Score of 3 C. extensive skin burns D. respiratory arrest

The answer is C. A lightning strike is the discharge of a massive amount of current over a very short period of time. This often causes "short-circuiting" of electrical systems such as heart, respiratory centers, and central and autonomic nervous systems, in addition to arterial and muscular spasm. However, significant skin burns and deep tissue destruction seldom occur.

The treatment of cardiogenic shock may include all of the following EXCEPT: A. Phenylephrine B. Treatment of ischemia C. Dobutamine D. Intra-aortic balloon pump E. Dopamine

The answer is A. The goal of therapy is to improve oxygenation, minimize ischemia, improve pump function, and decrease afterload. Dobutamine is the agent of choice in the setting of heart failure. An intra-aortic balloon pump may be a temporizing measure. Phenylephrine would increase afterload and worsen cardiac output.

A 28 year old man is shot in the right chest. His vital signs are: T 97.9, HR 120, BP 70/50, RR 12, SPO2 84%. He has significant JVD, his trachea is deviated to the left, and he has decreased breath sounds on the right. The patient's diagnosis is: A. Tension pneumothorax B. Spinal cord injury C. Hypovolemic shock D. Lung laceration E. Cardiac tamponade

The answer is A. The patient has a gunshot wound to the right chest near the sternum. His hypotension and JVD suggest an obstructive shock. Given the tracheal deviation to the left and absent breath sound on the right, he most likely has a tension pneumothorax. His emergent treatment includes needle decompression followed by chest tube placement.

A 15 year old boy dives into a swimming pool, hits his head on the bottom, and subsequently is found to have no sensation or motor function below the nipple line. His vital signs are: T 97.9, HR 76, BP 80/40, RR 12, SPO2 84%. He has no JVD and his lungs are clear. The patient's diagnosis is: A. Neurogenic shock B. Sepsis C. Spinal shock D. Near drowning E. Hypovolemic shock

The answer is A. The patient is most likely suffering a spinal cord injury, producing a disruption of the autonomic nervous system leading to vasodilation and hypotension (without the expected tachycardic response). This entity, called neurogenic shock, is a type of distributive shock like anaphylactic shock It is important to rule out other internal injuries in this patient, and then institute therapy with a pressor agent such as phenylephrine.

A thin 18 year old female complains of acute onset of sharp right-sided chest pain this morning. She has developed some mild shortness of breath during the morning and thought she should get it checked out. Her chest X-ray is shown in the Figure. The next course of action should be: [image] A. Chest tube placement B. Antibiotics C. Electrocardiogram D. Needle decompression

The answer is A. This patient has a large right-sided spontaneous pneumothorax that is not under tension. She needs oxygen and chest tube placement. This can be done with proper procedural analgesia and sedation since there is no immediate threat. Primary spontaneous pneumothorax tends to occur in healthy young men (and, less commonly, women) of taller than average height. Other risk factors include cigarette smoking, asthma, COPD, interstitial lung disease, connective tissue diseases, and lung cancers.

Treatment with hyperbaric oxygen (HBO) is associated with contraindications. Which of the following is not a relative or absolute contraindication to HBO? A. pregnancy B. untreated pneumothorax C. COPD with air trapping D. otitis media

The answer is A. Untreated pneumothorax is an absolute contraindication to HBO therapy. The reason is concern that it can progress to tension pneumothorax, especially during the decompression phase of therapy. The COPD patient with a large bleb represents a relative contraindication for similar reasons. Treatment with doxorubicin, and many other drugs -- such as cisplatin (Cisplatinum®), bleomycin (Blenoxane®), disulfiram (Antabuse®), and mafenide acetate (Sulfamylon®) -- contraindicates HBO therapy because of potentially toxic effects when combined with HBO. URI illnesses such as otitis media are relative contraindications, due to the potential for tympanic membrane rupture secondary to inability of the ears to equalize pressure during therapy. This can be addressed through myringotomy with placement of tubes (in cases where multiple HBO treatments are anticipated). In pregnant patients, HBO therapy has been shown to be safe for the fetus when given at appropriate levels and "doses" (durations). In fact, pregnancy lowers the threshold for HBO treatment of carbon monoxide-exposed pregnant patients. This is due to the high affinity of fetal hemoglobin for CO.

With respect to laboratory findings in diabetic ketoacidosis (DKA) and hyperglycemic hyperosmolar nonketotic coma (HHNC), all of the following guidelines are generally true EXCEPT: A. BUN is elevated more in patients with HHNC (>50 mg/dL) than in patients with DKA (25-50 mg/dL). B. Serum bicarbonate is typically severely low (<10mEq) in patients with either DKA or HHNC. C. Serum ketones are present in patients with DKA but not usually in patients with HHNC. D. Patients with HHNC typically have blood glucose > 700 mg/dL, whereas patients with DKA have blood glucose > 350 mg/dL. E. Serum osmolality in patients with HHNC is typically > 350 mOsm/L.

The answer is B. Although patients with DKA typically have profound metabolic acidosis with serum bicarbonate < 10mEq, acidosis is typically absent in patients with HHNC and serum bicarbonate is usually > 15 mEq.

Regarding the epidemiology of asthma in the United States, which of the following is true? A. Etiology is thought to be genetic, not environmental B. Prevalence increased in the 1980's, and then decreased in the 1990's C. More common in males than females in adult and pediatric populations D. Incidence is comparable for Caucasians and African-Americans

The answer is B. Despite an increase in asthma prevalence in the United States, Canada, Great Britain and Australia in the 1980s, the 1990s saw a decrease in prevalence in these areas. Regarding gender, male children are more likely than female children to have asthma, however the reverse is true with adults. African-Americans have a higher prevalence of asthma than Caucasians. Migrants who relocate from an area of low asthma prevalence to an area of high asthma prevalence tend to have an increased prevalence of asthma suggesting a role for environmental factors in the development of asthma.

Which of the following is not a common sign or symptom of thyrotoxicosis? A. congestive heart failure B. hypothermia C. hyperhidrosis D. nervousness E. tachycardia

The answer is B. Fever, not hypothermia, is commonly seen in thyrotoxicosis. Other common signs and symptoms include tachycardia, congestive heart failure, wide pulse pressure, tremor, thyrotoxic stare, thyromegaly, nervousness, weight loss, and palpitations.

Regarding the diagnosis and treatment of hypoglycemia, which of the following is correct? A. Glucagon, administered intramuscularly or subcutaneously, is a safe and universally effective means for increasing blood sugar in hypoglycemic patients. B. Hypoglycemia can present with virtually any neurological deficit. C. Hypoglycemia in adults is typically symptomatic at or below serum glucose of 60 mg/dL. D. Patients who overdose on oral hypoglycemic agents such as sulfonylureas must have their serum glucose monitored for a minimum of 6 hours before emergency department discharge. E. Patients with type 1 diabetes do not typically develop hypoglycemia.

The answer is B. Glucagon is ineffective in patients without adequate glycogen stores, as would be expected in alcoholics. Further, glucagon can precipitate a severe lactic acidosis in patients with glycogen storage diseases and therefore should not be used in children with hypoglycemia of unknown etiology. Typical symptoms of hypoglycemia include sweating, tachycardia, nervousness, hunger, and neurologic symptoms. Symptoms should not be attributed to hypoglycemia unless the level falls below 40-50 mg/dL. Type 1 diabetics practicing strict control of serum glucose are at high risk for hypoglycemic episodes precipitated by skipping a meal, or by increasing energy output or insulin dose. Due to the extended half-lives of the oral hypoglycemic agents, hospitalization and 24-hour observation (at minimum) are the typical management for overdose of these agents.

A 25-year old female presents to the ED with dyspnea and chest pain. Chest CT, with contrast, is performed and some pertinent "slices" are shown in the Figure. What is the diagnosis? [image] A. Aortic dissection, Type I B. Bilateral pulmonary embolism C. Gas embolism D. Acute Respiratory Distress Syndrome

The answer is B. Helical CT studies of the pulmonary vasculature are increasingly used for detection of pulmonary embolism. Though there are questions about CT's ability to detect small (e.g. subsegmental) emboli, CT scans have high sensitivity for proximal embolism such as that depicted in the accompanying figure. The patient whose images are shown was found to have moderate-severe right ventricular dysfunction and received thrombolytic therapy in the ED - she had an excellent outcome.

Symptoms of secondary adrenal insufficiency include all of the following EXCEPT: A. anorexia B. hyperpigmentation C. nausea and vomiting D. weight loss E. weakness

The answer is B. Hyperpigmentation is seen in greater than 90% of primary adrenal insufficiency. It is a result of compensatory adrenocorticotropic hormone (ACTH) and melanocyte-stimulating hormone (MSH) secretion. The secretion is a feedback mechanism that is not activated in secondary adrenal insufficiency, for example, adrenal insufficiency from pituitary infarction or hypothalamic insufficiency.

For a young otherwise healthy patient in anaphylactic shock, the initial best treatment of those listed below is: A. Broad spectrum antibiotics B. Epinephrine C. Steroids D. Diphenhydramine E. Dopamine

The answer is B. In a patient suffering from anaphylactic shock, the correct emergency treatment is epinephrine (especially in a young person in whom coronary disease is not a major concern). Steroids and diphenhydramine play a role in treatment of allergic reaction, but they are of a lower priority in anaphylaxis.

Which of the following is correct regarding the use of corticosteroids in acute asthma exacerbation? A. Beneficial effects occur within the first hour of administration. B. Inhaled steroids should be avoided C. Intravenous steroids are superior to the oral route D. Tapering is needed with all corticosteroid regimens

The answer is B. Oral and intravenous steroids are equally efficacious in treating an asthma exacerbation. Yet, in the setting of a severe asthma exacerbation, a patient may have difficulty taking oral medications and the intravenous route is preferred

A 44 year old male presents with fever, cough and shortness of breath. He is a nonsmoker. He has a lobar infiltrate on CXR. The most likely cause of pneumonia is: A. H. influenzae B. Strep pneumoniae C. Moraxella catarrhalis D. Mycoplasma pneumoniae E. viral

The answer is B. Streptococcus pneumoniae is the most common cause of community- acquired pneumonia. Viral is the second most common cause.

Regarding mycobacterium tuberculosis (TB), which of the following is TRUE? A. Worldwide, approximately 45-50% of the population is infected with TB B. Worldwide, TB causes more deaths than any other infectious agent C. In the United States, approximately 12-15% of the population is infected with TB. D. In the United States, the incidence of TB has been steadily increasing since the 1950s.

The answer is B. TB causes more deaths worldwide than any other infectious agent. Approximately one-third of the world's population is infected with TB. In the United States, only about 4-6% of the population is infected with TB. The incidence of TB infection in the United States began decreasing in the 1950s as a result of advancements in public health and pharmacologic treatments. Yet, the incidence has been rising since the 1980s, especially in poor, homeless, minority, and prison populations. The incidence of multi-drug resistant TB has been gradually increasing, especially among AIDS patients.

Regarding the diagnosis and treatment of thyroid storm in the emergency department, which of the following is true? A. A stat thyroid-stimulating hormone (TSH) level is required to make the diagnosis. B. Patients suspected of having thyroid storm should undergo treatment prior to a definitive diagnosis due to the potentially life-threatening nature of this disease. C. The diagnosis of thyroid storm is generally a straightforward clinical diagnosis and rarely confused clinically with other disorders such as psychiatric or other endocrine disorders. D. Thyroid storm cannot be diagnosed in the absence of altered mental status. E. Treatment of thyroid storm should only be undertaken after consultation with an endocrinologist.

The answer is B. The diagnosis of thyroid storm in the emergency department may be challenging due to the relatively infrequent occurrence of the disease and its typically nonspecific signs and symptoms. Treatment should be initiated in a timely fashion in any patient suspected of having thyroid storm due to the potential lethality of this disease. Immediate laboratory testing is typically not available to confirm clinically suspected cases, although thyroxine (T4) radioimmunoassay and free T4 index are good screening tests for hyperthyroidism. Clinical presentation of thyroid storm may be mistaken for psychiatric illness, heat stroke, sympathomimetic toxidromes, hypoglycemia and withdrawal syndromes, among others. Altered mental status, though frequently present, is not a prerequisite for diagnosis.

A 23 year old college novice mountain climber decides to climb a mountain with friends as a graduation present. His first night is spent at an altitude of 8,500 feet at a mountain resort. The next morning he starts to experience a mild headache and nausea. His symptoms get worse throughout the day. His friends want to get to 10,000 feet by nightfall. As a physician at the hotel, the worst advice you can give him is: A. Take supplemental oxygen. B. Ascend with the rest of the team. C. Take acetazolamide. D. Take ibuprofen.

The answer is B. The syndrome of high altitude illness ranges from mild AMS (Acute Mountain Sickness) to life threatening conditions of HAPE (High Altitude Pulmonary Edema) and HACE (High Altitude Cerebral Edema). This student is experiencing mild AMS. After the symptoms of altitude illness occur, further ascent to a higher sleeping altitude is contraindicated. Halting ascent or activity to allow further acclimatization may reverse symptoms. Acetazolamide is a carbonic anhhydrase inhibitor that induces a renal bicarbonate diuresis, causing a metabolic acidosis and thereby increasing ventilation and arterial oxygenation. Supplemental oxygen addresses the hypoxic insult of high altitude exposure. Ibuprofen is useful for the treatment of his headache. Dexamethasone can help with the symptoms of AMS, but does not play a role in acclimatization.

Of the choices below, the best treatment of the patient with hyperkalemia and EKG changes is: A. amiodarone B. defibrillation C. calcium D. vasopressin E. lidocaine

The answer is C. Hyperkalemia with EKG changes is treated with calcium to stabilize cardiac membranes. Calcium works quickly and is relatively safe unless patients are digitalized. Other treatments for acute hyperkalemia include sodium bicarbonate and insulin/glucose.

All of the following are true regarding the epidemiology of hypothyroidism EXCEPT: A. Approximately half of myxedema cases are diagnosed after admission to the hospital. B. Hypothyroidism occurs three to ten times more frequently in women than men. C. Hypothyroidism does not occur in infants under six months of age. D. Peak incidence of hypothyroidism is in the seventh decade. E. Most cases of hypothyroidism manifest in the winter months.

The answer is C. Hypothyroidism may occur at any age including the very young, but is infrequently seen in infants due to regular newborn screening for hypothyroidism. The increased frequency of the disease in women is attributed to the increased prevalence of autoimmune thyroid conditions in women. The majority of cases present in winter months due to the body's decreased ability to accommodate to cold weather in a hypothyroid state.

Which of the following is true regarding the use of iodine in the treatment of thyroid storm? A. Iodine should be administered even in patients with known iodine allergy. B. Iodine should be the first drug administered in the treatment of thyroid storm. C. Iodine should be administered at least one hour after propylthiouracil (PTU) has been given. D. Iodine should be administered only after treatment with propranolol. E. Dexamethasone must be given 30 minutes prior to iodine administration.

The answer is C. Iodine inhibits preformed thyroid hormone release and should be administered at least one hour after treatment with PTU to prevent organification of iodine. A typical dose is potassium iodide (SSKI) 5 drops every 6 hours PO or NG, or sodium iodide 1 gm slow IV drip every 8 to 12 hours. Iodine should not be administered to patients with known iodine allergy.

A 20 year old man was ice skating on a frozen pond and fell through the ice. The water was only about six feet deep and he was able to keep his head above water while bystanders were able to extract him after 10 minutes. The patient was transported to the emergency department and had an initial core temperature of 30 C. The patient's wet clothes were immediately removed and rewarming was initiated. Which of the following physical examination findings is expected? A. Hyperventilation B. shivering C. Altered mental status D. Tachycardia

The answer is C. Moderate hypothermia is associated with temperatures of 28-32 C. Shivering ceases at about 32 degrees Celsius. Moderate hypothermia is associated with altered mental status, absence of shivering, bradycardia, and bradypnea.

Which of the following drugs is MOST beneficial in an acute COPD exacerbation? A. Methylxanthines such as theophylline B. Steroids such as solumedrol C. Beta adrenergic agonists such as albuterol D. Mucokinetic agents such as acetylcysteine

The answer is C. Mucokinetic agents should not be used acutely in treatment of COPD exacerbation. These agents act to mobilize secretions, and this increases the work of the patient's breathing.

Which of the following is true regarding the treatment of acute asthma exacerbation in the Emergency Department? A. Intramuscular terbutaline is preferred over intravenous B. Heliox should only be used in the intubated patient. C. Anticholinergics by inhalation may be beneficial D. Intravenous albuterol may be indicated

The answer is C. Salmeterol is a long-acting beta2-selective adrenergic agonist that has no role in the treatment of an acute asthma exacerbation, but it is frequently preferred for outpatient asthma management due to its BID dosing schedule.

In the emergency department, regarding the disposition of patients diagnosed with pneumonia: A. Patients over 50 years of age should be admitted B. Patients under 2 years of age should be admitted C. Patients with co-morbidities should be admitted D. Patients with normal vital signs should be discharged home

The answer is C. The decision whether to admit or discharge patients diagnosed with pneumonia in the emergency department is not straightforward. Despite numerous guidelines including those recommending admission for patients with co-morbid disease such as HIV, CHF, malignancy, renal disease, liver disease and others, the decision ultimately resides in the clinical judgment of the emergency department physician. Likewise, though most patients with normal vital signs are typically well enough to be treated as outpatients, circumstances may exist in which this choice is not optimal, such as an elderly patient who lives alone and has poor follow-up.

All of the following are reasonable fluids for resuscitation of hypovolemia EXCEPT: A. Blood B. Lactated Ringer's C. D5W D. Albumin E. Normal saline

The answer is C. The goal of IV resuscitation is to restore intravascular volume. Fluids that are isotonic are preferred. D5W is hyptonic, and therefore a poor choice for volume resuscitation.

A 28 year old patient arrives after helicopter transfer from an outlying center, where he had been intubated for altered mental status after significant alcohol intoxication. There were no reported signs of chest trauma, but the patient now has decreased breath sounds on the left. His vital signs are stable. Based upon the chest X-ray in the figure, what is the next step in management of this patient? [image] A. Nasogastric tube placement B. Chest tube placement C. Endotracheal tube adjustment D. Needle decompression

The answer is C. The patient has a right-mainstem intubation and resultant opacification of the left lung secondary to unilateral lack of ventilation. In an adult male, the ETT should generally be inserted to a depth (to the lip line) of 22-24cm; the corresponding depth range for an adult female is 21-23cm

A 55 year old male, who has been missing for several days in wintertime, is found in a forested area several miles away from his house. He is brought to the ED where he is found to have a core temperature of 27 degrees Celcius. He clearly has diminished mental capacity. His initial ECG demonstrates atrial fibrillation with a ventricular rate of 110. Which of the following is the best treatment option? A. Start calcium channel blockage B. Apply a Bair Hugger C. Administer warm IV fluids D. Immerse in a warm water bath at 40 Celcius

The answer is C. The patient is suffering from severe hypothermia. Atrial dysrhythmias are common below 32o C and are associated with a slow ventricular response. It usually converts spontaneously with rewarming. While answers B through E are all active rewarming techniques (active external - Bair Hugger, AVA rewarming, immersion, active core - peritoneal lavage), the best answer for someone with severe hypothermia with mental status change and cardiac dysrhythmias is probably active core rewarming . This technique minimizes rewarming collapse in patients with temperatures below 32o C. The patient will likely need intubated as ileus, bronchorrhea, and depressed protective airway reflexes are common with hypothermia.

A 24 year old woman presents with difficulty breathing after eating Chinese food. Her vital signs are: T 97.9, HR 120, BP 80/40, RR 28, SPO2 86%. Her voice is hoarse and her lung auscultation reveals wheezes. She has no JVD. The patient's most likely diagnosis is: A. Acute myocardial infarction B. Pulmonary embolism C. Anaphylactic shock D. Spinal shock E. Cardiogenic shock

The answer is C. The patient's respiratory symptoms, absence of JVD, and vital signs suggest anaphylaxis (distributive shock). The proximity of symptoms to ingestion of food also suggests an allergic reaction.

A 79 year old man is brought into the emergency department by his two daughters for shortness of breath. They state that he has become increasingly short of breath over the last several weeks, and that he has been losing weight and sleeping more over the last several months. The patient says he feels fine and wants to know when he can have a cigarette. Physical exam reveals an oxygen saturation of 94% on room air and decreased breath sounds on the right. CXR is notable for a 3 x 4 cm opaque lesion in the right upper lobe that appears to be partially constricting the right mainstem bronchus. Which of the following is true regarding this man's condition and treatment? A. He should be urgently intubated due to the partial constriction of the right mainstem bronchus B. He should undergo urgent fluoroscopic-guided needle biopsy of the lesion to attain a diagnosis C. The lesion on CXR is worrisome for malignancy and warrants a chest CT D. Pulmonary angiogram is indicated to rule out pulmonary embolus

The answer is C. This man's symptoms of gradual onset of fatigue, weight loss and shortness of breath, as well as his history of cigarette smoking, are worrisome for a lung malignancy. The finding of mass on CXR supports this theory and is best further evaluated by chest CT. Although malignancy would put this man at risk for pulmonary embolus, a chest CT is also useful in working up pulmonary embolus and is typically used prior to pulmonary angiogram. Given that this man is not in severe respiratory distress at this point, there is no indication for intubation.

For a patient in neurogenic shock, the correct treatment would likely be all of the following EXCEPT: A. Phenylephrine B. High dose steroids C. Blood transfusion D. IV fluid bolus E. Spinal immobilization

The answer is C. Treatment of a spinal cord injury with neurogenic shock includes high dose steroids, IV fluids, immobilization, and potentially pressors. Blood transfusions are generally not indicated, and care must be taken to avoid fluid overload.

A 24 year old female gardener presents to the emergency department with foot pain 30 minutes after working barefoot in her garden. She saw a scorpion in the area. Which of the following signs or symptoms are most expected? A. Cranial nerve abnormalities B. Puncture mark C. Local erythema and swelling D. Pain and paresthesias

The answer is D. Although there are many toxic species of scorpions in the world, and all can sting humans, only a few cause serious toxicity. In the United States, only Centruroides exilicauda is capable of causing systemic toxicity. The sting is followed immediately by localized pain and paresthesias, and these can progress to involve the entire extremity or body. Systemic symptoms are unusual in adults, but more common and severe in children. Evidence of a sting, such as a puncture wound is almost never seen on exam. The mainstay of treatment is analgesia. Although antivenom is very effective in alleviating symptoms, both immediate and delayed allergic reactions occur with its use. Routine use of antivenom is not indicated, as most symptoms usually resolve in 1-2 days

A 7 year old girl is brought in by her father complaining of difficulty breathing after being stung by a bee. She is otherwise in good health. Neither she nor her father recall whether she has been stung previously, but state she has never before had a reaction like this. Which of the following treatments would be inappropriate? A. inhaled racemic epinephrine B. albuterol nebulizer C. heliox by face mask D. intravenous antibiotics

The answer is D. Antibiotics are not indicated for allergic reactions. Inhaled racemic epinephrine decreases inflammation of the airways and promotes bronchodilation. Heliox decreases turbulence of air flow to facilitate air movement into the airways. Albuterol nebulizer is a beta-agonist that causes bronchodilation, and intravenous cimetidine is an antihistamine that helps mitigate the allergic response.

A 65-year-old female presents with a chief complaint of palpitations and dyspnea on exertion. Vital signs are BP 130/84, HR 160 (and irregularly irregular), RR 14, T 37.8. EKG shows a narrow complex, irregularly irregular rhythm with absence of p-waves and an undulating baseline. What endocrine abnormality is most likely to be a direct cause of this abnormal rhythm? A. Hyperparathryoidism B. Addison's disease C. Cushing's syndrome D. Hyperthyroidism

The answer is D. Atrial fibrillation is a common arrhythmia. Its hallmark is the absence of P waves and irregular rhythm. It is associated with many medical conditions including ischemic heart disease and thyrotoxicosis. Atrial fibrillation increases the risk of thrombus formation and arterial embolism. AF's many treatment options include calcium channel blockers, beta blockers, amiodarone, quinidine, and cardioversion. Pacing is not a treatment option.

A radiographic finding that may be seen in an elderly patient who aspirated food is: A. Visualization of the food bolus in the right mainstem bronchus B. An opacity in the right middle lobe bronchus C. Tension pneumothorax on the right, with leftward mediastinal shift D. Hyperinflation of the left lung

The answer is D. Due to its wider diameter and the fact it forms a more obtuse take-off angle with the trachea, the right mainstem bronchus is a more likely site of aspiration than the left mainstem bronchus in adults. On chest X-ray, aspiration is more likely to be detected because of secondary signs, such as hyperinflation or atelectasis of the involved lung, rather than direct visualization of the aspirated contents. In this clinical scenario, the adult patient may have aspirated food into the right mainstem bronchus. One would therefore expect either atelectasis (collapse) or hyperinflation on the right side. The lobar bronchus to the right middle lobe has an acute take-off angle and poor collateral ventilation, which make it prone to atelectasis. Thus, it is the part of the right lung most likely to exhibit atelectasis after an aspiration event.

An 80 year old female presents to the ED with mental status changes after her neighbors found her this morning wandering in the stairwell. Patient was last seen normal 4 days ago and has no medical problems. On arrival to the ED, she is agitated and confused. Vital signs include RR of 20, HR of 100, BP of 90/50, and temperature of 40.6 Celsius. Pt is oriented to person only and is inattentive to exam, but appears to move all extremities symmetrically. She does not follow commands. Mucous membranes are dry and skin is dry and hot. What is the most likely diagnosis? A. Serotonin syndrome B. Encephalitis C. Thyroid storm D. Heat stroke

The answer is D. Heat stroke is a life-threatening illness defined clinically as a core body temperature that rises above 40.5 degrees Celsius and is usually accompanied by hot, dry skin (though in some cases sweating may be present) and central nervous system abnormalities such as delirium, convulsions, and coma. Treatment goals include lowering the core temperature to < 39.4 degrees Celsius by promoting cooling through conduction and evaporation and treating the complications that might arise with heat stroke, including seizures, respiratory failure, hypotension, rhabdomyolysis, and multi-organ dysfunction syndrome. Acute anticholinergic toxicity may sometimes present with a picture like that of heat stroke, and although the answer options C, D, and E may lead to confused behavior and/or fever in an elderly individual, the constellation of symptoms seen in this woman is most suggestive of heat stroke.

Which finding is commonly associated with pneumonia caused by Klebsiella pneumoniae? A. Absence of sputum B. Encephalitis C. Low mortality D. Empyema

The answer is D. Klebsiella pneumoniae can cause severe pneumonia, especially in the elderly or debilitated host. The mechanism of Klebsiella pneumoniae infection leads to a necrotizing, hemorrhagic process responsible for the classic "currant-jelly" sputum. Abscess formation, empyema and bacteremia are frequent complications, and the mortality rate is high.

Which of the following patients presenting to the emergency department with a chief complaint of "shortness of breath" is most likely to have cavitary lesion with air-fluid levels on CXR? A. 64 year old female with history of congestive heart failure B. 54 year old male with history of recent myocardial infarction C. 32 year old female who is 36 weeks pregnant D. 43 year old homeless male with history of alcohol abuse

The answer is D. The most common pathogens to generate cavitary lesions include tuberculosis, anaerobic bacteria (common pathogens in aspiration pneumonia), aerobic gram-negative bacilli, S. Aureus, and fungal disease. The patient described in C has multiple risk factors for developing a cavitary pneumonia, including a higher risk of tuberculosis from being homeless and a higher risk of aspiration pneumonia due to alcoholism. Congestive heart failure would cause a diffuse pattern of edema on CXR

A 4 year old girl is brought to the ED two hours after being stung by a scorpion while on a camping trip in Arizona. She has periods of agitation and restlessness alternating with calmness. Her vital signs are: blood pressure 106/61, pulse 120, respiratory rate 24, temperature 37.0C, and oxygen saturations of 99% on room air. On physical examination you note drooling, a disconjugate gaze, and occasional jerking movements of the extremities. Which of the following is the most correct regarding the treatment of a scorpion sting in this child? A. Treatment with antivemon is not indicated because these symptoms will be self-limiting B. Analgesics have a minimal role in controlling symptoms C. The patient should be intubated because respiratory failure is expected D. Complications of treatment with antivenom include delayed serum sickness

The answer is D. Most scorpion envenomations are mild, limited to pain and paresthesias at the site of envenomation. Children are affected more severely than adults: restlessness, jerking movements of the limbs, roving eye movements, and drooling are seen in severe cases. Anaphylaxis can also occur. Intubation is required rarely. Most envenomations require analgesics only; antivenom is indicated for severe reactions and anaphylaxis. Antivenom treatment is not without complications - serum sickness, and immediate and delayed hypersensitivity reactions occur. Without antivenom treatment, symptoms usually last for 1-2 days.

Which of the following patients is the most likely to develop S. pneumoniae pneumonia? A. 59 year old woman who is a cigarette smoker. B. 61 year old man with hypertension C. 65 year old woman with no past medical history D. 64 year old man with type 2 diabetes

The answer is D. Otitis media has not been associated with the development of S. pneumoniae pneumonia. The most common pathogen in community-acquired pneumonia, S. pneumoniae, is found in the nasopharynx of 40% of adults. Persons at risk for developing S. pneumoniae pneumonia include the elderly (>65), and those with a history of alcoholism, diabetes, cardiovascular disease, splenectomy, sickle cell disease, malignancy, and immunosuppressive disorders. Vaccination is recommended for all people at increased risk.

Regarding the development of cerebral edema in patients being treated for DKA, all of the following are true EXCEPT: A. Children have a higher incidence of cerebral edema. B. Patients with serum glucose below 250 mg/dL still being treated with insulin are most likely to develop clinically evident cerebral edema. C. Mortality of patients developing cerebral edema is 90%. D. Mannitol and steroids should be administered immediately to any patient suspected of developing cerebral edema. E. Cerebral edema typically occurs six to ten hours following onset of treatment.

The answer is D. Steroids are not indicated for treatment of cerebral edema and may actually worsen DKA. Mannitol 0.25-2.0 mg/kg should be administered upon any change in mental status of children being treated for DKA as they are at high risk for developing cerebral edema especially when being treated with insulin and serum glucose is below 250 mg/dL.

A 55-year-old female with a history of end-stage renal disease presents to the emergency department with weakness. Her electrocardiogram is shown below. What is her most likely diagnosis? [image] A. Acute MI B. Hypocalcemia C. Pericarditis D. Hyperkalemia

The answer is D. The EKG shows signs of hyperkalemia as characterized by diffuse peaked T waves. Other EKG changes include widening of the QRS complex and biphasic QRS-T segments. The heart rate may be slow, with ventricular fibrillation and cardiac arrest as the terminal events. Acute myocardial ischemia can be represented by hyperacute T waves as well, but in these cases the T wave changes are more likely to be focal (i.e. in an anatomical distribution corresponding to the area of threatened myocardium).

The X-ray in the figure indicates: [image] A. Need to withdraw the endotracheal tube from the mainstem B. A chest radiograph that was taken with the patient rotated C. Right upper lobe pneumonia D. Mediastinal shift due to fluid in the right hemithorax

The answer is D. The patient is not intubated. The pathology in the right hemithorax appears as hyperdensity, rather than air density (not a pneumothorax), and involves more than the right upper lobe.

A 5 year old male is bitten by a snake while playing along a ditch. The child is brought to the ED by his parents with complaint of fang marks to the right index finger. On physical exam, you note absence of swelling to the right hand or fingers. He does appear to have 2 small superficial fang marks, but no bleeding or oozing is present. Vital signs are normal. What is the next most appropriate step in the management of this patient? A. Admit for observation of potential compartment syndrome B. Administer prophylactic antibiotics with gram positive sensitivity C. Administer weight based antivenom in pediatric patients D. Discharge home in 8 hours if patient's exam remains unchanged

The answer is D. The patient is unlikely to suffer envenomation if he does not have any local or systemic symptoms in 8 hours. The dosage of antivenom is dependent on the degree of symptoms and children receive a proportionately higher dose compared to adults. Prophylactic antibiotics are not recommended

A 24 year old woman, brought by her sister, enters the emergency department. The 24 year old is writhing in pain, clutching her abdomen and shivering. Her sister states that the patient had a therapeutic abortion performed 3 days ago and has been having worsening abdominal pain ever since. The patient's vital signs are: T 103.4 F, HR 128, BP 104/72, RR 28, O2 saturation 100% in room air and she has marked lower abdominal pain and voluntary guarding. The most appropriate steps in treatment for this woman's condition include all of the following EXCEPT: A. broad-spectrum antibiotics B. urgent ob/gyn consult to facilitate rapid transport of patient to the operating room C. laboratory studies including basic chemistry, complete blood count with differential, coagulations studies, DIC panel, serum pregnancy test, and blood cultures D. high-dose steroids E. intravenous fluids

The answer is D. This patient is most likely suffering from a septic abortion in which retained products of conception developed a local infection. This infection has now spread systemically causing the systemic inflammatory response and potentially causing sepsis. Immediate intravenous fluids and broad-spectrum antibiotics are necessary; however, steroids are contraindicated in such a situation and could worsen the infection. The patient needs to be taken to the OR urgently to remove the retained products of conception.

Regarding the diagnosis of pulmonary embolism (PE), which is correct. A. Autospsy studies indicate that approximately 30% of patients who die in the hospital have undiagnosed PE B. By 80 years of age, one out of four people will develop deep venous thrombosis C. One-year mortality of PE is 25% even with early diagnosis and treatment D. Pulseless electrical activity in Emergency Department cardiac arrest victims is associated with PE in 33%

The answer is D. Up to two-thirds of patients with PE have no symptoms of DVT. PE is also found in about half of patients who have DVT but who do not have symptoms of PE. Thus, the presence or absence of DVT or symptoms of DVT does not correlate well with the finding of PE.

Regarding the treatment of suspected but not confirmed adrenal insufficiency, which of the following is most appropriate? A. withholding of steroids until confirmation of the diagnosis of adrenal insufficiency B. cosyntropin 0.25mg IV x 1 C. cortisone 100mg IM every 6 hours D. hydrocortisone 100mg IV every 6 hours E. dexamthasone 4mg IV every 6 hours

The answer is E. Dexamthasone is the treatment of choice in suspected but not confirmed adrenal insufficiency. It will not affect the serum cortisol level; therefore, it will not interfere with the diagnosis of adrenal insufficiency using the ACTH stimulation test. Administering cosyntropin, a synthetic form of ACTH, and measuring the serum cortisol levels typically perform the ACTH stimulation test. In confirmed adrenal insufficiency, hydrocortisone IV or cortisone IM are the treatments of choice.

Metabolic abnormalities often seen with hypothyroidism include all of thefollowing EXCEPT: A. anemia B. hyponatremia C. respiratory acidosis from hypoventilation D. hypercholesterolemia E. hyperglycemia

The answer is E. Hyperglycemia is not typically associated with hypothyroidism. Hypoglycemia may be present, but is unusual, and may suggest hypothalamic-pituitary involvement. Hyponatremia is common and corrects with thyroid replacement. Hypercholesterolemia to over 250 mg/dL is typical. A mild normochromic, normocytic anemia may be present, in addition to respiratory acidosis from hypoventilation.

A 25 year old female presents with fever, productive cough, and shortness of breath. Physical exam reveals T101, RR 24 and SpO2 of 94%. Her left tympanic membrane is inflamed and there are TM bullae. The chest X-ray reveals a large right middle lobe pneumonia. The organism most commonly associated with this type of presentation is: A. H. influenzae B. Moraxella catarrhalis C. Legionella pneumoniae D. Strep pneumoniae E. Mycoplasma pneumoniae

The answer is E. Mycoplasma pneumoniae is a common cause of pneumonia in young adults. It is classically associated with bullous myringitis.

which of the following eye drops will dilate the pupil? Answers: 1.pilocarpine 2.Pred Forte 3.proparacaine 4.sulfacetamide 5.tropicamide

Tropicamide (and homatropine) are muscarinic antagonists (mydriatic) and C/I in AACG, pilocarpine is mascarinic agonist (miotic), Pred Forte is prednisone, proparacaine is topical analgesic, sulfacetamide (antibiotic)

A patient with which of the following conditions classically presents with conjunctival injection (a "red" eye)? Answers: 1.central retinal artery occlusion 2.central retinal vein occlusion 3.retinal detachment 4.ultraviolet keratitis 5.vitreous hemorrhage

UV Keratitis

The pharmacologic interventions most likely to improve outcome when given in the field, as compared to those given upon arrival in the emergency department, include all of the following EXCEPT: Answers: 1.dextrose 2.epinephrine 3.albuterol 4.adenosine 5.diazepam

adenosine

Which of the following is FALSE concerning emergency intervention for traumatic emergencies encountered by EMS paramedics? Answers: 1.When short transport time is expected, use of pneumatic antishock garment appears to be associated with increased mortality in penetrating torso injuries. 2.Aggressive fluid resuscitation prior to surgical hemostasis is an absolute standard of care to minimize post-traumatic morbidity. 3.Severely injured patients require endotracheal intubation. 4.Intubating head injured patients may result in dental or soft tissue damage. 5.Increasing MAP to near normal levels may cause hemodilution and decreased oxygen saturation.

aggressive fluids prior to surgery

First-line interventions started by the emergency physician for suspected peptic ulcer disease (PUD) may include: Answers: 1.antacids 2.H2 blockers 3.proton pump inhibitors (PPIs) 4.stopping NSAIDs 5.all of the above

all

In dealing with the potential violent patient, the emergency physician should: Answers: 1.Approach the patient in a calm, controlled and professional manner 2.Assume that the strength of the doctor-patient relationship will ensure safety 3.Deal with the patient in a isolated room to protect the patient's privacy 4.Use a loud voice and threaten to call security if the patient becomes agitated

approach calmly

A 5 year old child was eating an almond when he experienced sudden, intermittent bouts of choking and wheezing. Assuming this child aspirated an almond, which of the following is most likely to be seen on chest X-ray? Answers: 1.A. Atelectasis of the affected lung 2.B. Diaphragmatic flattening of the non-affected lung 3.C. Foreign body in the shape of an almond 4.D. Hypoinflation of the non-affected lung

atelectasis of the affected lung

A 60 year old female presents with palpitations. Her EKG, shown below, reveals: Answers: 1.atrial flutter 2.ventricular tachycardia 3.sinus arrhythmia 4.atrial fibrillation

atrial flutter

A 60 year old male presents with new onset confusion. Which of the following suggests a functional, as opposed to an organic etiology? Answers: 1.abnormal vital signs 2.acute onset 3.auditory hallucinations 4.disorientation

auditory hal

The five diagnostic criteria for Ludwig's angina include all the following EXCEPT: Answers: 1.large amounts of visible pus in the submandibular space 2.bilateral cellulitis 3.presence of gangrene 4.sparing of the glandular tissues 5.spread of cellulitis by continuity, not by lymphatics

large amounts of visible pus in submandibular space

Contact lens use predisposes patients to which condition? Answers: 1.acute angle closure glaucoma 2.central retinal vein occlusion 3.corneal ulcer 4.hyphema 5.retinal detachment

corneal ulcer

A 45 year old man is brought to the E.D., with markedly altered mental status as reported by someone who stays with him at a homeless shelter. The patient is very confused and obtunded, and unable to provide a cogent history; the person who brought him to the E.D. notes the patient has a "drinking problem." The patient's vital signs are normal, except for a respiratory rate of 22. As he lays in the stretcher, his appearance is as depicted in the Figure. Of the following choices, which physical finding is most likely to be present on physical examination? Answers: 1.Homan's sign 2.anterior chest pain upon leaning forward, which is relieved by lying flat 3.dendritic rash on the posterior thorax, with a sentinel lesion noted on the left shoulder 4.upon elevation of the arms to 90-degrees (in 0-degrees abduction), and pronation of the hands with fingers spread, wrists and interphalangeal joints are characterized by jerky alternations of extension and flexion 5.increase in systolic blood pressure by more than 10 mmHg with inspiration

elevation of hands jerky alternations

A 63 year old female presents to the ED at noon, stating that she noticed marked facial swelling (see Figure - top half) upon awakening that morning. She has breast cancer, without brain metastases on a recent MRI. She has no urticaria or respiratory symptoms. A CT scan of the chest was performed from the ED (see Figure - bottom half). Regarding this patient's condition, which of the following is true? Answers: 1.Seizures are a common presenting sign. 2.The vascular component of this problem commonly poses a threat to the patient's airway integrity. 3.Schizophreniform disorder is present when a patient meets the diagnostic criteria for schizophrenia but the process has been present for less than one year 4.Elevation of the head of the patient's bed is recommended.

elevation of patients head

Which of the following groups has an increased risk of ingested foreign body? Answers: 1.Asthmatics 2.Diabetics 3.Edentulous 4.Smokers

endentulous

A 58 year old woman presents to the emergency department one day after her cat bit her index finger. Physical examination shows signs of flexor tenosynovitis. She is admitted to the hospital for IV antibiotics, hand elevation, and emergent hand surgery consultation to consider debridement of the finger. Which of the following would be least likely expected in a patient with flexor tenosynovitis? Answers: 1.tenderness over the flexor tendon sheath 2.swelling of the finger 3.pain with passive extension 4.erythema of the flexor surface of the involved digit

erythema of flexor surface

A 19-year-old G1P0 female, at 38 weeks EGA, presents to the emergency department complaining of headache, blurry vision and leg swelling. The physical examination reveals BP of 150/100, facial and hand edema, and hyperreflexia. Fetal heart monitoring demonstrates a reassuring pattern with no uterine contractions. Urine dipstick reveals 2+ proteinuria. Of the following, which factor is the most critical in formulating an ultimate management plan for this patient? Answers: 1.Extent of maternal edema 2.Fetal age 3.Presence of variable decelerations on nonstress testing 4.Symptom duration

fetal age

A 54 year old male comes to the emergency department complaining of intermittent pain, swelling and constant burning sensation involving his right leg. He tells you that six months ago he injured his leg in a car accident and his x-rays were negative. The symptoms have gradually worsened over the past few months. He is presenting to the emergency department because he doesn't have a primary care doctor. On physical examination you notice that his leg is edematous, erythematous, dry and warm. The leg is also characterized by hair loss, allodynia and hyperesthesia. Of the following, which is the most appropriate emergency department course? Answers: 1.Arrange follow-up for presumed complex regional pain syndrome 2.Obtain a CT scan of his leg to rule out osteomyelitis 3.Order a venogram to rule out a deep venous thrombosis 4.Perform a femoral nerve block to control his pain

follow up CRPS

Delirium is defined as: Answers: 1.a stressed psychological state resulting from extreme emotional stimulus 2.abnormal behavior associated with decreased alertness and decreased psychomotor activity 3.abnormal behavior accompanied by hallucinations, occurring in an oriented patient 4.a global inability to relate to the environment and process sensory input

global inability

In treating hepatitis secondary to alcoholic liver disease, all of the following are true EXCEPT: Answers: 1.Treatment is primarily supportive including fluids and electrolyte correction. 2.Glucose should be administered prior to thiamine to avoid precipitating acute Wernicke's encephalopathy. 3.Magnesium replacement should be initiated empirically except in the setting of contraindications such as renal failure or hypermagnesemia. 4.Coexisting gastritis should be sought out and treated appropriately. 5.Nutritional status should be assessed with attention to possible protein restriction.

glucose prior to thiamine

The differential diagnosis of papilledema includes all of the following EXCEPT: Answers: 1.hydrocephalus 2.hypertensive encephalopathy 3.hyphema 4.intracranial mass 5.pseudotumor cerebri

hyphema, no papilledema, anterior chamber blood

A 55 year-old male presents with new onset agitation and confusion. Which of the following medical histories would suggest a psychiatric (non-organic) cause? Answers: 1.History of diabetes mellitus only 2.History of alcohol abuse only 3.History of hypothyroidism only 4.History of chronic obstructive pulmonary disease only

hypothyroidism

Recurrent cellulitis in the distal phalanx of the right thumb in a 32 year old carpenter who is otherwise healthy should prompt the emergency room physicians to perform: Answers: 1.A. A deep, longitudinal 2-cm incision to explore the thumb for an abscess 2.B. Radiographic imaging for suspected retained foreign body 3.C. Screening for Human Immunodeficiency Virus 4.D. Testing for diabetes mellitus

imaging for suspected retained body

A 53 year-old known alcoholic presents with agitation, vomiting and altered mental status. His fingerstick glucose is 148. His serum ethanol level is undetectable and his head CT is normal. An ABG shows a pH of 7.21, pCO2 of 34, pO2 of 98 on room air. His basic chemistry panel includes a sodium of 136, potassium 4.1, chloride 108, bicarbonate 14, BUN 12, creatinine 1.1. What substance are you concerned that he may have ingested Answers: 1.Ethylene glycol 2.Salicylates 3.Isopropyl alcohol 4.Methanol

isopropyl alcohol

A 32-year-old G1P0 woman at 32 weeks gestation presents to the emergency department complaining of a worsening headache. Her vital signs are T 98.4, BP 160/115, P 95, R 16. Her urinalysis reveals 3+ protein. Which of the following is the first choice agent to decrease her blood pressure? Answers: 1.fenoldopam 2.labetolol 3.nitroglycerin 4.nitroprusside 5.phentolamine

labetolol, nitroprusside

With regard to U.S. Emergency Medical Services (EMS), all the following are true EXCEPT: Answers: 1."First responders" are not always Emergency Medical Technicians (EMTs). 2.The levels of EMT training and EMT-level nomenclature are the same throughout the United States. 3.The Department of Transportation is the federal government agency tasked with promulgation of EMT training requirements. 4.A community is said to have "E-911" when the telephone number of a 911 caller is displayed at the operator's console. 5.Most EMTs in the field operate under off-line medical control.

levels of emt training

A 35 year-old male is placed on his back on the gurney in physical restraints for violent behavior. Which life-threatening complication can arise? Answers: 1.circulatory obstruction 2.Metabolic acidosis 3.Asphyxia 4.Rhabdomyolysis

metabolic acidosis

A 27 year old is found unresponsive in his car in the hospital parking lot and brought in by security. During your initial evaluation you find him to be cyanotic with pulse oximetry reading 82% on room air with a respiratory rate of 4 breaths per minute. Radial pulses are present at 120 bpm. Pupils are 1mm bilaterally. Your team is having difficulty finding a vein for an intravenous line due to extensive scarring of his arms. You are suspicious of an overdose, which medication would you want to rapidly administer as a potential antidote in this situation? Answers: 1.Glucose 2.Naloxone 3.Thiamine 4.Flumazenil

naloxone

A 28 year old female is 4 months pregnant and presents with dysuria. Her UA reveals leukocyte esterase and nitrates. Of the following, the best treatment is: Answers: 1.Ciprofloxacin 2.Macrodantin (nitrofurantoin) 3.Bactrim 4.Amoxicillin 5.Doxycycline

nitrofurantoin

A patient with a history of difficult-to-control hypertension is now 6 to 8 weeks pregnant and presents with a hypertensive emergency. Which of the following IV antihypertensives should be avoided? Answers: 1.nitroprusside 2.hydralazine 3.esmolol 4.labetolol 5.nicardipine

nitroprusside

Horner's syndrome is characterized by all of the following EXCEPT: Answers: 1.anhidrosis 2.enophthalmos 3.miosis 4.ophthalmoplegia 5.ptosis

opthalmoplegia; Horners miosis, pstosis, anhidrosis, and enopthalmos (posterior displacement of eyeball, apparent due to eyelid droop)

A patient presents to the emergency department with alcohol intoxication (precluding a useful history) and eye pain. The extrnal eye exam reveals an eliptically misshapen pupil, and a hyphema is also noted. A slice from a head CT (obtained due to patient's unclear history and intoxication). Of the following choices, which is the best next step for the emergency physician evaluating this patient? Answers: 1.A. palpation of the globe with mild digital pressure, to determine if increased intraocular pressure is present 2.B. performance of emergency lateral canthal tendon release 3.C. ophthalmology consultation for intraocular foreign body 4.D. topical corneal anesthesia followed by gentle sweep of a cotton swab over the eye to remove corneal foreign body

optho consult for intraocular foreign body

A 70 year old male with acute delirium requires administration of haloperidol for agitation. Which of the following is a recognized side effect of haloperidol? Answers: 1.first degree heart block 2.nephrogenic diabetes insipidus 3.prolonged QT interval 4.transient hepatitis

prolonged QT

If the parents are present and refuse treatment for their child in a life-threatening emergency, prehospital care providers should: Answers: 1.Confirm the identities of the parents and follow parental wishes 2.Provide treatment for the child 3.Contact on-line medical control for physician permission to treat 4.Contact their ambulance service's legal counsel to discuss whether to treat 5.Call the police to have the parents arrested

provide treatment

A pediatric patient is brought in by his mother, who notes he's had persistent nasal drainage. Plain films for sinusitis are obtained. Of the choices listed, which is the best next step for the patient? Answers: 1.A. Admission for IV antibiotics and oral decongestants 2.B. Discharge on antibiotics 3.C. MRI to further assess the sinuses 4.D. Removal of foreign body.

removal of foreign body

A patient presents with a self-inflicted wound, with resultant loss of vision in the right eye. With regard to the figure, which of the following statements is most likely true? Answers: 1.The patient will probably recover visual function. 2.Medial canthotomy should be performed immediately. 3.Search for other self-inflicted injuries (or ingestions) is paramount. 4.If ambulance providers contact medical control about a patient with this injury, they should be directed to replace the globe back into the orbit. 5.Life-threatening hemorrhage is a major risk with this injury.

search self inflicted

A "BLS" ambulance differs from an "ALS" ambulance in that the BLS ambulance: Answers: 1.is staffed by one EMT crew member (and one driver) rather than two EMTs 2.is a smaller "van"-type ambulance 3.arrives at the patient first 4.operates under off-line, as opposed to on-line, medical control 5.is stocked with different supplies and equipment

stocked different

A 54 year old male presents with complaints of a foreign body sensation in his left eye. Which of the following is an indication of urgen ophthalmologic referral? Answers: 1.A. You discover a moderate-sized linear corneal abrasion 2.B. You note a small puncture wound in the globe 3.C. Your examination reveals a metallic foreign body that is loosely adherent to the white of the eye 4.D. You note an intraocular pressure of 20 in the affected eye

small puncture in globe

Management of a 4 year old child with a two-week history of malodorous purulent nasal drainage should include, as an initial step: Answers: 1.A. Oral amoxicillin for 10-14 days 2.B. Referral to an otolaryngologist 3.C. Social services consultation to investigate potential child abuse 4.D. Speculum examination of the nares

speculum exam of nares

All of the following are common pathogens in otitis media EXCEPT: Answers: 1.Strep pneumoniae 2.H. influenzae 3.Staph. aureus 4.viral agents 5.Moraxella catarrhalis

staph aureus

A patient presents with itching and swelling in the leg, which is depicted in the Figure, which shows both the posterior thigh and an insect removed from the thigh. Which of the following is true regarding this patient's management? Answers: 1.Use of forceps in an attempt to remove this insect is contraindicated 2.This insect rarely transmits multiple diseases simultaneously 3.To be exposed to this insect, the patient probably has traveled out of the U.S. within the last month 4.There is a vaccine available for U.S. use, which is active against a disease transmitted by this insect

there is a vaccine available for lyme disease though rarely used

Regarding the EMS role in prehospital care, all the following are true EXCEPT: Answers: 1.Survival after cardiac arrest is less than 10% when resuscitation efforts are initiated after 10 minutes from the arrest. 2.Thrombolytic therapy in the field is the standard of care for patients suspected of having an acute coronary syndrome. 3.Pharmacotherapy that can be initiated in the field by paramedics in most jurisdictions includes naloxone for opiate overdose, diazepam for seizure control, and beta agonists for acute asthma exacerbations. 4.Automatic external defibrillators (AEDs) can be used by firefighters, policemen and other trained first responders. 5.EMS personnel evaluate and release many patients they deem well enough not to need hospital treatment.

tpa standard of care

All of the following are common non-infectious causes of fever EXCEPT: Answers: 1.Neuroleptic malignant syndrome 2.Trauma 3.CVA 4.Thyroid storm 5.Pulmonary embolism

trauma

Regarding the diagnosis of pyelonephritis, which of the following is FALSE? Answers: 1.Patients with pyelonephritis typically have symptoms for greater than 5 days. 2.White blood cell casts on urinalysis support a diagnosis of pyelonephritis. 3.Abnormal vaginal discharge is typically seen on pelvic exam in patients with pyelonephritis. 4.Diabetic patients with bacteriuria are at increased risk for developing pyelonephritis. 5.Pyelonephritis is more common in indigent populations.

vaginal discharge


Conjuntos de estudio relacionados

GEOM A, U4L5: Lines and Triangles Practice

View Set

Ch 19: Heart and Neck Vessels (2 sets)

View Set

Med-Surg Chapter 52: Concepts of Care for Patients With Inflammatory Intestinal Disorders

View Set

Key Concepts of Computer Technology

View Set

9 Physical and Cognitive Development in Middle Childhood

View Set

Consumer Behavior Exam 1 (Chapter 3)

View Set

Chapter 5 - Technology and Its Effects

View Set

introduction, scientific method, history, sleep, drugs, brain, bio

View Set